23.2 Flashcards

1
Q

A patient who underwent a thoracotomy 6 months ago reports ongoing pain caused by light brushing of clothes against the skin on the chest wall. This is known as

a) Hyperalgesia
b) Allodynia
c) Hyperaesthesia
d) dysasthesia

A

Mechanical allodynia

Allodynia IASP definition: pain due to a stimulus that does not normally provoke pain

“The term allodynia was originally introduced to separate from hyperalgesia and hyperesthesia, the conditions seen in patients with lesions of the nervous system where touch, light pressure, or moderate cold or warmth evoke pain when applied to apparently normal skin.”

References IASP https://www.iasp-pain.org/resources/terminology/?ItemNumber=1698
And APMSE 5th Ed pg64.

Dysaesthesia: spontaneous and unpleasant sensation

How well did you know this?
1
Not at all
2
3
4
5
Perfectly
2
Q

According to Australian and New Zealand Committee on Resuscitation (ANZCOR) guidelines, during advanced life support for ventricular fibrillation, adrenaline 1mg should be administered

a) As soon as possible
b) Before shock
c) After 2nd shock
d) After 3rd shock

A

C.

Shockable:
Adrenaline 1mg after 2nd shock
Then every second cycle
Amioderone 300mg after 3 shocks

Non-shockable
Adrenaline 1mg immediately
(then every second cycle)

How well did you know this?
1
Not at all
2
3
4
5
Perfectly
3
Q

The Sequential Organ Failure Assessment (SOFA) score is used in intensive care for the
assesment of sepsis. This score does NOT include the:

a) MAP
b) FiO2/PaO2
c) INR
d) GCS
e) Plts

A

Previous Q (23.1) with different options.
ANSWER C (INR)

How well did you know this?
1
Not at all
2
3
4
5
Perfectly
4
Q

In an adult patient with reduced mouth opening, insertion of a classic design LMA may be easier than with other supraglottic airways because of its

a) Bite block
b) Gastric port
c) Low profile
d) Preformed curve

A

a) low profile

Resource:
ANZCA PG56(A)BP Difficult airway equipment BP 2021

First generation SADs (page 19)
“classic design LMAs (cLMAs) with their low profile and lack of preformed curve have several advantages.”

How well did you know this?
1
Not at all
2
3
4
5
Perfectly
5
Q

You are asked to assess a patient in the intensive care unit who has a tracheostomy that may have become dislodged. To assess if the tracheostomy is patent you should NOT

a) Put in a bougie
b) Suction cath
c) Deflate cuff
d) Remove speaking valve
e) Remove inner cannula

A

A

High risk of creating a false passage

Blue book 2017 page 21

No reference

How well did you know this?
1
Not at all
2
3
4
5
Perfectly
6
Q

Albumin is contraindicated in

a) Traumatic brain injury

A

No remembered options.
Answer could be:
Traumatic Brain injury
Direct allergy
Cardiac Failure

SAFE trial

How well did you know this?
1
Not at all
2
3
4
5
Perfectly
7
Q

A bleeding patient has ROTEM results including (ROTEM results shown). The most
appropriate treatment is

a) Plts
b) FFP
c) Cryo
d) TXA

A

c) Cryo

Cryo or TXA,

TXA first line treatment however patient has low fibrinogen and requires fibrinogen replacement.

How well did you know this?
1
Not at all
2
3
4
5
Perfectly
8
Q

A 56 year old patient presents with exertional syncope. The most likely diagnosis is

a) HOCM
b) Long QT
c) CCF
d) Myocardial ischaemia

A

Repeat: 20.2

HOCM if these remembered options are correct

Alternative is Aortic Stenosis which is more common than HOCM in this age group

As per Cardiology

How well did you know this?
1
Not at all
2
3
4
5
Perfectly
9
Q

The shoulder joint receives sensory innervation from all of the following nerves EXCEPT the

a) Axillary
b) Long thoracic
c) Lateral pectoral nerve
d) Supra scapular
e) Sub scapularis

A

b) Long thoracic

How well did you know this?
1
Not at all
2
3
4
5
Perfectly
10
Q

The most likely diagnosis for the following electrocardiograph is

a) VF
b) AF w bundle branch block
c) SVT w BBB
d) VT
e) Sinus w BBB

A

d) VT

https://litfl.com/ventricular-tachycardia-monomorphic-ecg-library/

How well did you know this?
1
Not at all
2
3
4
5
Perfectly
11
Q

The nerve labelled with an arrow in the diagram below (diagram of lumbar plexus shown) is the

a) Obturator
b) Accessory obturator
c) Genitofemoral
d) Ilioinguinal
e) Iliohypogastric

A

Lumbar plexus questions already exist in combined deck

How well did you know this?
1
Not at all
2
3
4
5
Perfectly
12
Q

A pregnant woman requires a caesarean section delivery within 30 minutes for fetal distress.

Her body mass index (BMI) is 26 kg/m2. She has multiple sclerosis with lesions in her brain and spinal cord and receives monthly injections of the disease-modifying drug ofatumumab.

The most appropriate plan for her delivery is

a) Spinal
b) GA
c) CSE
d) Epi

A

a) Spinal
Makarla

Epidural and vaginal delivery
? GA

all are safe in MS
The MAN I think is to signify advanced MS

(Really there isn’t heaps of evidence)

Source World Fed Anaesthetists

https://resources.wfsahq.org/wp-content/uploads/359_english.pdf
(What a terrible question)

How well did you know this?
1
Not at all
2
3
4
5
Perfectly
13
Q

The needle whose tip is pictured is a

a) Sprotte
b) Quinke
c) Touhy
d) Whitacre

A

c) Touhy

How well did you know this?
1
Not at all
2
3
4
5
Perfectly
14
Q

Patients with rheumatoid arthritis and the most common form of atlantoaxial instability have a widened atlantodental interval. This is measured between the

A. distance from posterior surface of dens to anterior surface of posterior arch of atlas
B. distance from anterior surface of dens to anterior surface of posterior arch of atlas
C. distance from posterior surface of dens to anterior surface of anterior arch of atlas
D. distance from posterior surface of dens to posterior surface of posterior arch of atlas
E. distance from anterior surface of dens to posterior surface of anterior arch of atlas

A

Repeat 23.1

E. distance from anterior surface of dens to posterior surface of anterior arch of atlas

The atlantodental interval is used in the diagnosis of atlanto-occipital dissociation injuries and injuries of the atlas and axis.

The anterior atlantodental interval is the horizontal distance between the posterior cortex of the anterior arch of the atlas (C1) and the anterior cortex of the dens in the median (midsagittal) plane

Normal values for anterior atlantodental interval are:
radiographs:
adults:
males: <3 mm
females: <2.5 mm 1 (although most authors describe <3 mm ref)

children:
<5 mm ref

CT: adults: <2 mm

How well did you know this?
1
Not at all
2
3
4
5
Perfectly
15
Q

Abdominal compartment syndrome is defined by the presence of end-organ dysfunction with a lower limit of abdominal pressure measured at

A. 10mmHg
B. 16mmHg
C. 20mmHg
D. 24mmHg

A

repeat 23.1

c) 20mmHg

Intra-abdominal hypertension is defined as a sustained intra-abdominal pressure of >12 mm Hg, and abdominal compartment syndrome occurs at a pressure >20 mm Hg in association with new organ dysfunction.

Intra-abdominal hypertension is graded as follows: Grade 1=12–15 mm Hg; Grade 2=16–20 mm Hg; Grade 3=21–25 mm Hg; and Grade 4 >25 mm Hg.

How well did you know this?
1
Not at all
2
3
4
5
Perfectly
15
Q

Pulmonary hypertension is defined as a mean pulmonary arterial pressure greater than
a) 15mmHg
b) 20mmHg
c) 25mmHg
d) 30mmHg

A

b) 20mmHg

How well did you know this?
1
Not at all
2
3
4
5
Perfectly
16
Q

Of the following, the patient characteristic associated with an increased risk of developing severe bone cement implantation syndrome is

a) Female
b) Diuretics
c) Young age
d) Previous orthopaedic instrumentation
e) B blocker

A

b) Diuretics

Grade 1: moderate hypoxia (Spo2<94%) or hypotension [fall in systolic blood pressure (SBP) >20%].

Grade 2: severe hypoxia (Spo2<88%) or hypotension (fall in SBP >40%) or unexpected loss of consciousness.

Grade 3: cardiovascular collapse requiring CPR.

Patient Risk factors:
1. old age
2. poor pre-existing physical reserve
3. impaired cardiopulmonary function
-> NYHA 3 or 4
4. pre-existing pulmonary htn
5. Male Sex
6. Diuretics
7. ASA grade 3 or 4
8. osteoporosis
9. bony metastases
10. concomitant hip fractures (particularly pathological and intertrochanteric)
(latter due to abnormal vascular channels through which marrow contents can enter the circulation)

Surgical Risk factors
1. patients with previously un-instrumented femoral canal > revision surgery
2. Use of long-stem femoral component

Anaesthetic Risk reduction:
- discussion between surgeons and anaesthetists over uncemented vs. cemented based on patient Hx particularly if lon-stem prosthesis, femoral fracture or patients with cardiorespiratory disease
- no clear evidence regarding the impact of anaesthetic technique
- increase inspired O2 considered in all patients at time of cementation
- avoid intravascular volume depletion
- Higher level of haemodynamic monitoring in high risk patients

Factors NOT predictive of severe BCIS include:

Arteriosclerosis
Angina pectoris
Congestive heart failure
Beta-blockers
Angiotensin-converting enzyme inhibitors.
How well did you know this?
1
Not at all
2
3
4
5
Perfectly
16
Q

An inappropriate irrigation solution when using monopolar diathermy during transurethral resection of prostate would be

a) 1.5% Glycine
b) 5% dextrose
c) 3% Mannitol
d) 0.9% Saline
e) Sorbitol

A

d) 0.9% Saline

Other fluids are all electrolyte free except 0.9% Saline

How well did you know this?
1
Not at all
2
3
4
5
Perfectly
17
Q

The technique of airway pressure release ventilation

a. Has a prolonged expiratory time
b. Augments cardiac output in hypovolaemic patients
c. Results in reduced mean airway pressures

A

none of the remembered options

Airway pressure release ventilation (APRV) is an open-lung mode of invasive mechanical ventilation mode, in which spontaneous breathing is encouraged.
APRV uses longer inspiratory times; this results in increased mean airway pressures, which aim to improve oxygenation.
Brief releases at a lower pressure facilitate carbon dioxide clearance.
The terminology and methods of initiation, titration, and weaning are distinct from other modes of mechanical ventilation.
The use of APRV is increasing in the UK despite a current paucity of high-quality evidence

high intrathoracic pressure decreases the transmural left ventricular pressure, reducing the work of contraction and increasing cardiac output. In the context of hypoxaemia, a mode of mechanical ventilation that improves arterial oxygenation will improve myocardial oxygen delivery, myocardial function and cardiac output. As APRV is a spontaneous breathing mode, in addition to the benefits of spontaneous ventilation, reduced doses of sedative drugs can often be used, with subsequent reduction of requirement for vasoactive drugs and improvement in haemodynamic state.

Airway pressure release ventilation (APRV) is an open-lung mode of invasive mechanical ventilation mode, in which spontaneous breathing is encouraged. APRV uses longer inspiratory times; this results in increased mean airway pressures, which aim to improve oxygenation

https://www.bjaed.org/article/S2058-5349(19)30178-7/fulltext

https://derangedphysiology.com/main/required-reading/respiratory-medicine-and-ventilation/Chapter%20518/airway-pressure-release-ventilation-aprv-ards

How well did you know this?
1
Not at all
2
3
4
5
Perfectly
18
Q

The modified Aldrete scoring system uses all of the following EXCEPT the

a) BP
b) Pain score
c) Resp rate
d) sedation level

A

b) Pain score

Aldrete score, which includes five elements (activity, respiration, circulation, consciousness, oxygen saturation) [16].

The original scoring system was developed before the invention of pulse oximetry and used the patient’s colouration as a surrogate marker of their oxygenation status. A modified Aldrete scoring system was described in 1995 which replaces the assessment of skin colouration with the use of pulse oximetry to measure SpO2.

The Modified Aldrete system includes five additional elements that are particularly useful during the Phase II recovery period prior to discharge to home (dressing, pain, ambulation, feeding, urine output)

How well did you know this?
1
Not at all
2
3
4
5
Perfectly
19
Q

The ventilator waveforms shown represent (actual image from exam)

a) Triggered breaths
b) Bronchospasm
c) Obstructive pattern
d) Gas trapping

A

C) Obstructive Pattern

https://thoracickey.com/ventilator-graphics/
Image 9.6

How well did you know this?
1
Not at all
2
3
4
5
Perfectly
20
Q

An absolute contraindication to transoesophageal echocardiography is

A. Dysphagia
B. GORD
C. Oesophageal stricture
D. oesophageal webbing
E. oesophageal varices

A

C. Oesophageal stricture

https://www.asecho.org/wp-content/uploads/2014/05/2013_Performing-Comprehensive-TEE.pdf

How well did you know this?
1
Not at all
2
3
4
5
Perfectly
21
Q

According to the ATACAS trial, the continuation of low-dose aspirin prior to cardiac surgery is associated, in the postoperative period, with

a) No increased risk of bleeding
b) Decreased risk of MI
c) Increased risk of Thrombotic events
d) Increased risk of seizures

A

a) No increased risk of bleeding

There is no evidence that pre-operative aspirin administration resulted in a lower risk of death or thrombotic complications, or a higher risk of haemorrhage.

The study aim (and title) was to compare stopping vs continuing aspirin, however the design insisted on all patients stopping aspirin and then being given a single dose of aspirin or placebo prior to surgery (and presumably all patients were given aspirin after surgery) – this method hasn’t really investigated the theory

TheBottomLine.org.uk

How well did you know this?
1
Not at all
2
3
4
5
Perfectly
22
Q

A 43-year-old man is undergoing an elective endovascular coiling procedure for an 8 mm
middle cerebral artery aneurysm. Midway through the procedure the interventionalist tells you they have ruptured the aneurysm. All of the following are appropriate initial
interventions EXCEPT

A. Decrease BP
B. Give protamine
C. Urgent transfer to theatre
D. Continue coiling
E. Mild hyperventilation

A

REPEAT

Answer: c. Urgent transfer to theatre

BJA Anaesthesia for interventional neuroradiology
https://academic.oup.com/bjaed/article/8/3/86/293346

Clinical signs of a rise in ICP or a sudden rise in blood pressure with or without a fall in heart rate should alert the anaesthetist to this possibility. Extravasation of contrast may also be seen. The goals are to increase coagulability by reversing heparin, decrease bleeding by lowering blood pressure (to the level before the bleed), control ICP with hyperventilation, head elevation, steroids and osmotic agents, control seizures, and initiate cerebral protection. Once the bleeding is controlled, the pressure may be raised to check for leaks. Usually, the coiling continues; rarely, a ventriculostomy may be required. If the coiling is unsuccessful, a rescue craniotomy and clipping will be required. Management may also involve performance of CT scans and subsequent transfer to ICU.

How well did you know this?
1
Not at all
2
3
4
5
Perfectly
23
Q

A patient will open her eyes in response to voice, speak with inappropriate words and
withdraw to a painful stimulus. Her Glascow Coma Scale score is

A

E3 V3 M4 = GCS 10

How well did you know this?
1
Not at all
2
3
4
5
Perfectly
24
Q

Intravenous dexmedetomidine use does NOT result in

a) hypotension
b) Unchanged PACU length of Stay
c) residual sedation
4) Reduced in pain

A

c) residual sedation

https://pubmed.ncbi.nlm.nih.gov/35085107/#:~:text=Conclusions%3A%20The%20use%20of%20dexmedetomidine,sedation%20or%20bradycardia%20in%20PACU

How well did you know this?
1
Not at all
2
3
4
5
Perfectly
25
Q

The risk of developing postherpetic neuralgia may be reduced by treating acute herpes zoster (shingles) with

A. Ibuprofen
B. Gabapentin
C. Aciclovir
D. Amitriptyline
E. Oxycodone

A

D. Amitriptyline

Amitriptyline (used in low doses for 90 days from onset of the herpes zoster rash) reduces the incidence of postherpetic neuralgia

N.B
Antiviral agents started within 72 hours of onset of the herpes zoster rash accelerate the resolution of acute pain (U) (Level I) but do not reduce the incidence, severity and duration of postherpetic neuralgia

UTD
Both Gabapentinoids and TCAs are effective at TREATING postherpetic neuralgia. The former have lower risk of discontinuation due to adverse side effects.
For moderate or severe pain, use gabapentinoids.

How well did you know this?
1
Not at all
2
3
4
5
Perfectly
26
Q

Rapid reversal of the anticoagulant effect of dabigatran can be achieved with

a) Andexenet Alfa
b) rotuzimab
c) Idarucizumab (Praxbind)
d) Infliximab

A

Idarucizumab (Praxbind) is a monoclonal antibody to dabigatran

Dabigatran bleeding may be treated with:
- idarucizumab
- haemodialysis
-PCC 25-50IU/kg
- TXA will decrease fibrinolysis and has some effect
- FFP also has some effect

Humanized monoclonal antibody fragment (Fab) indicated in patients treated with dabigatran (Pradaxa) when reversal of the anticoagulant effects are needed for emergency surgery or urgent procedures, or in the event of life-threatening or uncontrolled bleeding
- very high affinity for dabigatran (300x vs affinity for thrombin)
- 5 g IV, provided as 2 separate vials each containing 2.5 g/50 mL (see Administration)
- RE-VERSE-AD trial: undetectable levels <20ng/ml within minutes and for 24 hours
- Limited data support administration of an additional 5 g depending on clinical situation

Dosage Modifications

Renal impairment: Renal impairment did not impact the reversal effect of idarucizumab; no dosage adjustment required
Hepatic impairment:
Dosing Considerations

This indication is approved under accelerated approval based on a reduction in unbound dabigatran and normalization of coagulation parameters in healthy volunteers; continued approval for this indication may be contingent upon the results of an ongoing cohort case series study

How well did you know this?
1
Not at all
2
3
4
5
Perfectly
27
Q

The muscle or muscle group with the greatest sensitivity to the action of non-depolarising neuromuscular blocking agents is/are the

a. Abdominal muscles
b. Adductor pollicis
c. Pharyngeal muscles
d. Diaphragm

A

c. Pharyngeal muscles

Millers Anaesthesia:
Reference artyicle from Millers: https://pubs.asahq.org/anesthesiology/article/92/4/977/710/The-Incidence-and-Mechanisms-of-Pharyngeal-and

An adductor pollicis TOF ratio of 0.90 or less was associated with impaired pharyngeal function and airway protection, resulting in a four- to fivefold increase in the incidence of pharyngeal dysfunction causing misdirected swallowing. Moreover, pharyngeal function and airway protection may be impaired, even if the adductor pollicis muscle has recovered to a TOF ratio of more than 0.90.

How well did you know this?
1
Not at all
2
3
4
5
Perfectly
28
Q

A patient under general anaesthesia monitored with transcranial cerebral oximetry has a decrease in their cerebral oxygen saturation. This is likely to be improved by an increase in all of the following EXCEPT

A. Increasing blood pressure
B. Deepening anaesthesia
C. Increased minute ventilation
D. Transfusion

A

C. Increased minute ventilation

Cerebral blood flow
Cardiac output
Acid–base status
Major haemorrhage
Arterial inflow/venous outflow obstruction

Oxygen content
Haemoglobin concentration
Haemoglobin saturation
Pulmonary function
Inspired oxygen concentration
Inspired oxygen concentration

How well did you know this?
1
Not at all
2
3
4
5
Perfectly
29
Q

Based on this tracing (single ECG lead shown), the mode in which this pacemaker is operating is:

a) VVI with intermittent failure to capture
b AVI with failure to captue
c) AVI with failure to sense
d) VVI with failure to sense
e) VVD

A

e) VVI with intermittent failure to sense

How well did you know this?
1
Not at all
2
3
4
5
Perfectly
29
Q

Elimination of remifentanil occurs following breakdown mainly by

a Plasma cholinesterase
b RBC esterases
c Hoffman degradation
d Hepatic Metabolism
e Plasma esterases

A

e Plasma esterases

Plasma esterases (not cholinesterase)

Esmolol metabolism is via RBC esterases.

How well did you know this?
1
Not at all
2
3
4
5
Perfectly
30
Q

A patient who has had a previous axillary nodal dissection and who does not have
lymphoedema of the affected arm presents for surgery. On the affected arm

A

Check ANZCA documents

How well did you know this?
1
Not at all
2
3
4
5
Perfectly
31
Q

Synchronised direct current cardioversion is NOT indicated when the arrhythmia is

a) AF
b) Flutter with rate <100
c) Multifocal atrial tachy
d) SVT with
e) Conscious torsades

A

C- Multifocal Atrial Tachycardia

Cardioversion is contraindicated in MAT. Due to the multiple atrial foci, direct current (DC) cardioversion is not effective in restoring normal sinus rhythm and can precipitate more dangerous arrhythmias.
- https://emedicine.medscape.com/article/155825-overview#a10

DCCV is indicated for
1. Any haemodynamically unstable narrow or wide QRS complex tachycardia
2. AF <48hrs
3. AF >48hrs with adequate anticoag/TOE to exclude thrombus
4. SVTs and monomorphic TVs not responding to trial of IV medical therapy

DCCV is CONTRAindicated in:
a. Digitalis toxicity and associated tachycardia
b. AF >48hrs without adequate anticoagulation/TOE
-BJAEducation 2017
https://academic.oup.com/bjaed/article/17/5/166/2669966

How well did you know this?
1
Not at all
2
3
4
5
Perfectly
32
Q

In a 20-year-old with cystic fibrosis, the most likely finding on pulmonary function tests is

a. Mixed obstruction and restrictive pattern
b. Restrictive with normal DLCO
c. Restrictive with low DLCO
d. Obstruction with reduced RV
e. Obstructive with reduced FEV1

A

REPEAT 23.1

e. Obstructive w/ reduced FEV1

Mucous narrowing airways = obstructive
Parenchymal damage = restrictive

Obstructive PFP remains the most common pulmonary function pattern in adult CF and is associated with
-decrease FEV1 & FVC/FEV1

For patients with CF, an obstructive pattern is generally seen, with a decrease in forced expiratory volume in 1 s (FEV1), and forced vital capacity (FVC) to FEV1 ratio.

https://academic.oup.com/bjaed/article/11/6/204/263786

How well did you know this?
1
Not at all
2
3
4
5
Perfectly
33
Q

Dulaglutide reduces blood glucose by

A - Binding Glucagon-like peptide 1 receptors and causing activation
B - Binding Glucagon-like peptide 1 receptors and competitively inhibiting GLP1 binding
C - Binding Glucagon-like peptide 1 receptors and causing conformational change leading to cell death
D - Binding L cells of the gastrointestinal mucosa leading to GLP-1 secretion
E - Binding L cells of the gastrointestinal mucosa leading to GLP-1 sequestration

A

A - GLP1 receptor agonist
(rest of options made up)

“Dulaglutide binds to glucagon-like peptide 1 receptors, slowing gastric emptying and increases insulin secretion by pancreatic Beta cells. Simultaneously the compound reduces the elevated glucagon secretion by inhibiting alpha cells of the pancreas, as glucagon is known to be inappropriately elevated in diabetic patients. GLP-1 is normally secreted by L cells of the gastrointestinal mucosa in response to a meal”
- Wikipedia, Dulaglutide
- Once weekly injection, “trulicity”

https://www.asahq.org/about-asa/newsroom/news-releases/2023/06/american-society-of-anesthesiologists-consensus-based-guidance-on-preoperative

How well did you know this?
1
Not at all
2
3
4
5
Perfectly
34
Q

Ongoing cerebral seizure activity induced by electroconvulsive therapy should be medically terminated after

a) 30 seconds
b) 1 minute
c) 2 minutes
d) 3 minutes
e) 5 minutes

A

c) 2 minutes

How well did you know this?
1
Not at all
2
3
4
5
Perfectly
35
Q

Characteristics of post-operative visual loss due to vertebrobasilar ischaemia include

a) inattention
b) Vision returns in 24hrs
c) relevant afferent pupillary defect
d) diplopia

A

d) diplopia

Bilateral visual loss associated with insufficiency to posterior circulation so: parieto-occipital ischaemia, signs of stroke, visual agnosia, ophthalmoplegia or diplopia.

How well did you know this?
1
Not at all
2
3
4
5
Perfectly
35
Q

The odds ratio is the measure of choice for a

a. Case control
b. Cohort
c. RCT
d. Epidemiological study

A

a) case control

https://www.cdc.gov/csels/dsepd/ss1978/lesson3/section5.html

How well did you know this?
1
Not at all
2
3
4
5
Perfectly
36
Q

The diagnostic criterion for severe obstructive sleep apnoea in adults is an apnoea/hypopnoea index of at least

A) 10
B) 20
C) 30
D) 40
E) 50

A

C) 30

How well did you know this?
1
Not at all
2
3
4
5
Perfectly
37
Q

When used for prolonged analgesia in a healthy adult, the recommended maximum dose of ropivacaine via continuous infusion or bolus dosing in a 24-hour period is

a) 450mg
b) 600mg
c) 770mg
d) 1200mg

A

c) 770mg

Product info: Fresenius-Kabi

When prolonged epidural blocks are used, either by continuous infusion or repeated bolus administration, the risks of reaching a toxic plasma concentration or inducing local neural injury must be considered. Cumulative doses of up to 800 mg ropivacaine for surgery and postoperative analgesiaadministered over 24 hours were well tolerated in adults, as were postoperative continuous epidural infusions at rates up to 28 mg/hour for 72 hours.

product info: pfizer

When prolonged blocks are used, either through continuous infusion or through repeated bolus administration, the risks of reaching a toxic plasma concentration or inducing local neural injury must be considered. Experience to date indicates that a cumulative dose of up to 770 mg ropivacaine hydrochloride administered over 24 hours is well tolerated in adults when used for postoperative pain management: i.e., 2016 mg. Caution should be exercised when administering ropivacaine for prolonged periods of time, e.g., > 70 hours in debilitated patients

How well did you know this?
1
Not at all
2
3
4
5
Perfectly
38
Q

In a cardiac transplant recipient, hypotension due to general anaesthesia is least likely to respond to

a) noradrenaline
b) Ephedrine
c) adrenaline
d) Atropine

A

d) Atropine

Blue book 2019

How well did you know this?
1
Not at all
2
3
4
5
Perfectly
39
Q

When the infraclavicular approach is used, the brachial plexus is blocked at the level of the

a. roots
b. trunks
c. divisions
d. cords
e. branches

A

d. cords

How well did you know this?
1
Not at all
2
3
4
5
Perfectly
40
Q

A patient’s glomerular filtration rate is estimated at 35 mL/min/1.73m2. The patient’s chronic kidney disease can be classified as Stage
a) 2
b) 3a
c) 3b
d) 4
e) 5

A

c) 3b

How well did you know this?
1
Not at all
2
3
4
5
Perfectly
41
Q

Maintaining a CO2 pneumoperitoneum at a pressure of 15 mmHg is most likely to lead to

a) Lactic acidosis
b) Decreased arterial blood pressure
c) Decreased heart rate
d) Increased CVP
e) Increased renal blood flow
f) Increased SVR

A

REPEAT

f) Increased SVR

How well did you know this?
1
Not at all
2
3
4
5
Perfectly
41
Q

The Myocardial Injury after Non Cardiac Surgery study showed elevated troponin in the first three post-operative days was strongly associated with

a) 30 day mortality
b) 30 day MI

A

New question.

a)

Postoperative myocardial injury was associated with an increased risk of death. Twenty-seven of the 315 patients (8.6%; 95% CI, 6.0–12.2%) with myocardial injury died within 30 days compared with 29 of the 1312 patients (2.2%; 95% CI, 1.5–3.2%) with normal troponin I levels (P<0.01)

Reference: Myocardial Injury After Noncardiac Surgery and its Association With Short-Term Mortality (Circulation 2013)

How well did you know this?
1
Not at all
2
3
4
5
Perfectly
41
Q

The National Audit Project 6 found that the most common early clinical feature of perioperative anaphylaxis was

a) Arrest
b) Urticaria
c) Bronchospasm
d) Hypotension
e) CO2 down

A

d) Hypotension

The commonest presenting feature of perioperative anaphylaxis by far was hypotension (accounting for 46%), followed by bronchospasm/high airway pressure (18%), tachycardia (9.8%), flushing/non-urticarial rash 6.6% and cyanosis/oxygen desaturation (4.7%).

How well did you know this?
1
Not at all
2
3
4
5
Perfectly
42
Q

You are inducing anaesthesia in a 20-year-old female through a cannula which was inserted in the right antecubital fossa while she was in the emergency department. After 10 ml of propofol has been injected, she complains of severe pain and it becomes clear that the cannula is intra-arterial. The most appropriate management is

a) aspirate
b) flush with N.Saline
c) flush with lignocaine
d) observe
e) flush with Heparin

A

https://www.anztadc.net/Publications/Images/ANZCA/Unintended%20Intraarterial%20injection%20WebAIRS%20news%20ANZCA%20Bulletin%20September%202019.pdf

How well did you know this?
1
Not at all
2
3
4
5
Perfectly
43
Q

A patient with a history of restless leg syndrome is agitated in the post-anaesthesia care unit.
After excluding other causes, the best treatment of the agitation in this patient is

a) Pethidine
b) Clonidine
c) Droperidol
d) Haloperidol
e) Midazolam

A

midazolam

  • Opioids, benzodiazepines and pregabalin may also be used to alleviate symptoms.

Perioperative treatment of symptoms
If RLS symptoms occur perioperatively, patients should be allowed to walk or move their legs in bed as soon as possible.
If prolonged bed rest is required, the frequency of RLS medications may be increased to three times a day.
If oral intake is feasible, a patient’s usual oral medication may be given.
Levodopa (a dopamine agonist) may be administered by nasogastric tube.
Alternatively, parenteral apomorphine or a rotigotine patch may be used.
Apomorphine (1 milligram) may be injected subcutaneously on an hourly basis.
Nausea is a common side effect so it may need to be given with an antiemetic.
Rotigotine patches may be used every 24 hours.
Opioids, benzodiazepines and pregabalin may also be used to alleviate symptoms.
Patients should be proactively investigated and treated for iron deficiency, targeting ferritin level greater than 300 micrograms/ litre in adults, and 50 micrograms/litre in children.

How well did you know this?
1
Not at all
2
3
4
5
Perfectly
43
Q

A man with a history of obesity and obstructive sleep apnoea has just had a transsphenoidal pituitary resection. Soon after extubation he is semi-conscious and is making a respiratory effort but has near complete upper airway obstruction with stridor. His arterial oxygen saturation is 93% and starting to fall. Your first actions should be to

a) Deepen with propofol and insert LMA
b) Insert Oropharyngeal airway and provided positive pressure ventilation
c) Insert Nasopharyngeal airway and provided positive pressure ventilation
d) Insert Nasopharyngeal airway and provide CPAP

A

a) Deepen with propofol and insert LMA

Nasal continuous positive airway pressure (CPAP) is contraindicated after transsphenoidal surgery due to the risk of tension pneumocephalous. The level of consciousness, eye movements, visual fields, and acuity should be tested frequently and any deterioration discussed with the surgeon, and radiological investigation and/or re-exploration considered.

https://academic.oup.com/bjaed/article/11/4/133/266875#3195876

How well did you know this?
1
Not at all
2
3
4
5
Perfectly
44
Q

Measures to avoid venous air embolism when inserting an internal jugular central venous catheter in an awake patient include all of the following EXCEPT

a) Trendelenburg position
b) Occlude needle hub with thumb
c) Insert during inspiration
d) Pre-insertion IV fluid bolus

A

New question

Measures to avoid venous air embolism when inserting an internal jugular central venous catheter in an awake patient include all of the following EXCEPT

c) Insert during inspiration

Negative pressure generated by inspiration in an AWAKE patient

https://www.ncbi.nlm.nih.gov/pmc/articles/PMC5126790/

How well did you know this?
1
Not at all
2
3
4
5
Perfectly
45
Q

Diffusing capacity of the lungs for carbon monoxide (DLCO) is decreased in all of the following EXCEPT

made up potential answers:

a) Pulmonary Fibrosis
b) Interstitial Lung disease
c) Obesity
d) Pulmonary haemorrhage

A

d) Pulmonary haemorrhage

Rewording of 21.2 Question

Won’t increase in Myasthenia Gravis

Causes of HIGH value include:
Asthma
Left-right intracardiac shunt
polycythaemia
Pulmonary haemorrhage
Obesity - Dlco will increase but kco will not

How well did you know this?
1
Not at all
2
3
4
5
Perfectly
46
Q

A patient who is day 3 post laparotomy has used 30 mg oxycodone intravenously via patient controlled analgesia in the last 24 hours. The approximate oral morphine equivalent daily
dose is

a) 30mg
b) 45mg
c) 60mg
d) 90mg

A

90mg PO morphine

Oral Tapentadol 25mg = 8mg Oral Morphine

Oral Oxycodone 5mg = 8mg Oral Morphine

Oral Tramadol 25mg = Oral Morphine 5mg

Oral Hydromorphone 4mg = Oral Morphine 20mg

S/L Buprenorphine 200mcg = 8mg Oral Morphine

IV Oxycodone 5mg = Oral Morphine 15mg

IV Morphine 5mg = Oral Morphine 15mg

IV Hydromorphone 1mg = Oral Morphine 15mg

How well did you know this?
1
Not at all
2
3
4
5
Perfectly
46
Q

The recommended dose of IV adrenaline in a 15 kg, 5 year old child with grade 2 (moderate) perioperative anaphylaxis is

a) 15mcg
b) 30mcg
c) 50mcg
d) 100mcg
e) 150mcg

A

b) 30mcg

Moderate = 2mcg/kg
Life threatening = 4-10mcg/kg

file:///Users/jbjon/Downloads/Australian_and_New_Zealand_Anaesthetic_Allergy_Gro.pdf

How well did you know this?
1
Not at all
2
3
4
5
Perfectly
47
Q

A 21-year-old patient with a history of schizophrenia on quetiapine develops tremor,
restlessness, hyperreflexia, nausea and vomiting in the post-anaesthesia care unit following an emergency laparoscopic cholecystectomy. Her heart rate is 80 / minute, blood pressure 130/90 mmHg, and her temperature is 37.0°C. The most likely diagnosis is

a) Serotonin Syndrome
b) NMS
c) MH
d) Rhabdomyolysis
e) anticholinergic crisis

A

Repeat 22.2

Serotonin syndrome

Hyperreflexia differentiates

Usually has hypertension and hyperthermia

How well did you know this?
1
Not at all
2
3
4
5
Perfectly
48
Q

The success rate of stopping smoking before surgery is NOT improved by

a) Bupropion
b) Clonidine
c) Nortroptyline
d) Varencicline
e) SSRI

A

Repeat

SSRI

Clonidine has limited efficacy

ANZCA PG12 Background Paper

ANZCA PERIOP CESSATION OF SMOKING GUIDELINE:
“Effective pharmacotherapy options include nicotine replacement therapy, nicotine
partial agonists such as varenicline (Champix), bupropion (Zyban), nortryptilline and clonidine”

Up to Date Pharmacotherapy for Smoking Cessation in Adults
- First-line pharmacotherapies for smoking cessation include nicotine replacement therapy (NRT), varenicline, and bupropion
- Clonidine: despite promising initial studies, clonidine is now generally regarded as having limited efficacy for smoking cessation.
- Selective serotonin reuptake inhibitors/anxiolytics – Selective serotonin reuptake inhibitors (SSRIs) and anxiolytic drugs generally have not been shown to be effective for smoking cessation

How well did you know this?
1
Not at all
2
3
4
5
Perfectly
49
Q

The condition for which you would have a lower arterial oxygen saturation target is

Made up responses

a) Carbon monoxide poisoning
b) Bronchopulmonary Pneumonia
c) Bleomycin toxicity
d) Pulmonary Fibrosis

A

Answers could also possibly be COPD, Acute stroke or Neonates

c) Bleomycin toxicity

Bleomycin is a particularly important chemotherapy drug for the anaesthetist to be aware of. Bleomycin is often used to treat germ cell tumours and Hodgkin’s disease in a curative setting. The major limitation of bleomycin therapy is the potential for subacute pulmonary damage that can progress to life-threatening pulmonary fibrosis. Pulmonary toxicity occurs in 6–10% patients and can be fatal.2 Exposure to high-inspired concentration oxygen therapy, even for short periods, as experienced during anaesthesia, is often implicated in causing rapidly progressive pulmonary toxicity in patients previously treated with bleomycin.3 These claims have been considered controversial by some, but it is the authors’ recommendation that any patient previously exposed to bleomycin therapy should be treated as high risk, and summary guidance regarding oxygen therapy is shown in Table 4.

Summary guidance—oxygen therapy for patients who have received bleomycin > Patients have a life-long risk of bleomycin-induced lung injury
> Oxygen therapy should be avoided if at all possible
> Clinical procedures (and leisure activities) involving a high should be avoided If a patient is hypoxic
> O2 therapy should be minimized to maintain O2 saturation of 88–92%
> High oxygen concentrations should be used with extreme caution for immediate life-saving indications only (to maintain O2 saturation of 88–92%)

How well did you know this?
1
Not at all
2
3
4
5
Perfectly
49
Q

In the thigh, the adductor canal is bordered by all of the following EXCEPT

a) Adductor Longus
b) Adductor Magnus
c) Sartorius
d) Vastus Lateralis
e) Vastus Medialis

A

d) Vastus Lateralis

Anteromedial: sartorius
Lateral: vastus medialis
Posterior: adductor longest and magnus

How well did you know this?
1
Not at all
2
3
4
5
Perfectly
50
Q

When performing cannulation of the median cubital vein the structure that is LEAST likely to be inadvertently punctured or damaged is the

A) Radial artery
B) Median nerve
C) Brachial artery
D) Ulnar artery
E) Ulnar nerve

A

Repeat

e) Ulnar nerve

The cubital fossa is triangular in shape and consists of three borders, a roof, and a floor:
Lateral border – medial border of the brachioradialis muscle.
Medial border – lateral border of the pronator teres muscle.
Superior border – horizontal line drawn between the epicondyles of the humerus.
Roof – bicipital aponeurosis, fascia, subcutaneous fat and skin.
Floor – brachialis (proximally) and supinator (distally).
Contents:
- radial nerve
- biceps tendon
- brachial artery
- median nerve

Mnemonic for contents of the cubital fossa:
Really Need (radial nerve) Beer To (biceps tendon) Be At (brachial artery) My Nicest (median nerve).

51
Q

A patient has severe hypokalaemia and is in cardiac arrest. The Australian Resuscitation Council and the New Zealand Resuscitation Council recommend intravenous potassium should be given as

a) 5mmol bolus KCl
b) 5mmol KCl over 5min
c) 5 mmol KCl over 10min
d) 10mmol bolus KCl
e) 20mmol KCl over 10min

A

REPEAT

a) 5mmol bolus KCl

https://www.anzcor.org/home/adult-advanced-life-support/guideline-11-5-medications-in-adult-cardiac-arrest/

Potassium is an electrolyte essential for membrane stability. Low serum potassium, especially in conjunction with digoxin therapy and hypomagnesaemia, may lead to life threatening ventricular arrhythmias.

Consider administration for:
Persistent VF due to documented or suspected hypokalaemia.
[Class A; Expert consensus opinion]
Adverse effects:
Inappropriate or excessive use will produce hyperkalaemia with bradycardia, hypotension and possible asystole

Extravasation may lead to tissue necrosis.

Dosage:
A bolus of 5 mmol of potassium chloride is given intravenously.

52
Q

You wish to place the tip of a central venous line at the cavo-atrial junction in an adult, which on a chest X-ray is at a level

A

Repeat

52
Q

In the POISE study the use of beta blockers on the day of surgery as a cardio protective strategy in high risk patients has been associated with

a) Increased heart rate
b) Decreased hypotension
c) Increased mortality
d) Increased myocardial infarction

A

REPEAT
c) Increased mortality

Use of perioperative metoprolol was associated with:
* Decreased rate of myocardial infarction
* Decreased rate of revascularisation
* Decreased rate of developing new atrial fibrillation
* INCREASED rate of death
* INCREASED rate of stroke
* INCREASED rate of significant hypotension
INCREASED rate of significant bradycardia

53
Q

Following denervation injury to muscles, critical hyperkalaemia associated with suxamethonium administration can occur as early as

a) 12hrs
b) 18hrs
c) 24hrs
d) 48hrs

A

d) 24hrs

Extrajunctional receptors are not found in normal active
muscle but appear very rapidly whenever muscle activity has
ended or after injury has been sustained. They can appear
within 18 h of injury and an altered response to neuromuscu-
lar blocking drugs can be detected within 24 h of the insult.
They disappear when muscle activity returns to normal.

54
Q

A 25-year-old man suffers a burn involving 30% of his total body surface area. A
cardiovascular physiological change expected within the first twenty-four hours is

a. Decreased PVR
b. Increased SVR
c. Decreased SVR
d. Reduced PA pressure
e. Increased hepatic blood flow

A

REPEAT

increased SVR

EMSB handbook
CO is reduced after Burn injury 2ry to:
- myocardial depressant mediators
- decreased blood volume
- reduced venous return
- increased pulmonary and systemic vascular resistance due to increased levels of catecholamines

In the first 24hrs reduced cardiac output persists even after restoration of blood volume

Between 24-48hrs post burn a hyperdynamic state develops with reduced peripheral resistance, increased oxygen consumption and increased cardiac output

55
Q

For an adult patient with septic shock, the 2021 Surviving Sepsis Guidelines suggest using
procalcitonin to guide

a) Start/stop steroids
b) Stop antibiotics
c) Start CRRT
d) Source control

A

b) Stop/stop antibiotics

For adults with suspected sepsis or septic shock, we suggest AGAINST using procalcitonin plus clinical evaluation to decide when to start antimicrobials, as compared to clinical evaluation alone.
Weak, very low quality of evidence

For adults with an initial diagnosis of sepsis or septic shock and adequate source control where optimal duration of therapy is unclear, we suggest using procalcitonin AND clinical evaluation to decide when to discontinue antimicrobials over clinical evaluation alone.
Weak, low quality of evidence

56
Q

ANZCA guidelines recommend that under general anaesthesia, blood pressure should be
measured no less frequently than every

a) 2 mins
b) 3 mins
c) 5 mins
d) 10 mins

A

10mins
PG18

57
Q

Suxamethonium is safe to use for muscle relaxation in a patient with

a. Becker muscular dystrophy
b. Myaesthenia gravis (new option)
c. Guillain Barre
d. Hypokalaemic periodic paralysis (new option)
e. Duchenne muscular dystrophy

or

a. Becker muscular dystrophy
b. Cerebral palsy
c. Guillain Barre
d. Frederich’s ataxia
e. Duchenne muscular dystrophy

A

b. Myaesthenia gravis or b. Cerebral palsy

ED95 is 0.8mg/kg in a MG patient

b. Cerebral palsy
->sux and volatiles are not contraindicated
-> presence of extrajunctional receptors may cause hyperkalaemia

a. Becker muscular dystrophy
-> essentially milder Duchenne’s (see duchenne response to Sux)

b. Cerebral palsy
-> Sux and volatiles not contraindicated
-> reduced MAC requirement
-> increased sensitivity to muscle relaxants

c. Guillain Barre
-> sux contraindicated due to risk of hyperkalaemia
-> increased sensitivity to Non depolarising NB

d. Frederich’s ataxia
-> sux should be avoided due to risk of hyperkalaemia

e. Duchenne muscular dystrophy
-> sux and volatiles contraindicated due to rick of hyperkalaemia and rhabdomyolysis

58
Q

The changes in oximetry seen after intravenous injection of indocyanine green are

A

REPEAT
Increases NIRS , decreases peripheral spo2

SctO2 up, SpO2 down.

Source: Korean Journal Anaesthesia
https://www.researchgate.net/publication/274570990_Effects_of_intravenously_administered_indocyanine_green_on_near-infrared_cerebral_oximetry_and_pulse_oximetry_readings

59
Q

Regarding cardiopulmonary exercise testing before major surgery, oxygen pulse is the

a. Arterial oxygen content at peak HR
b. Arterial oxygen saturation at mean HR?
c. Arterial oxygen saturation at peak HR
d. PaO2 at peak HR
e. Oxygen consumption/min divided by HR

A

REPEAT

e. Oxygen consumption/min divided by HR

VO2/HR: oxygen consumption divided by HR, known as the ‘oxygen pulse’ (ml beat–1)

https://www.bjaed.org/article/S2058-5349(19)30021-6/fulltext

The objective of CPET is to determine functional capacity in an individual.
Deficiencies in CPET-derived variables—specifically:
1. ventilatory anaerobic threshold (AT)
2. peak O2 consumption (VO2peak)
3. ventilatory efficiency for carbon dioxide (VE/VCO2)
—are associated with poor postoperative outcomes (mortality, morbidity, admission to intensive care, and length of hospital stay) after intra-abdominal surgery.

  1. Does the oxygen pulse increase with exercise?
    The oxygen pulse is the VO2 divided by HR, and represents the product of the stroke volume and the arterial-venous oxygen difference. It can be seen in panel 2 and can be viewed as a surrogate for stroke volume, and as such should increase at the start of exercise before slowly reaching a plateau at its highest predicted value.
59
Q

Appropriate surgical anaesthesia with sevoflurane is characterized by a frontal EEG showing

a) Decreased alpha and delta waves
b) Increased alpha waves
c) anteriorisation alpha waves
d) Increased gamma and epsilon
e) increased spectral edge frequency

A
60
Q

The medical laser LEAST likely to cause eye injury is

a) CO2
b) Nd:YAG
c) Argon
d) Green light

A

REPEAT

CO2

Laser danger is proportional to penetration.
Penetration inversely proportional to the laser wavelength.

CO2 laser has very little penetration (~ 10micrometres), as it has a wavelength of 10 600nm.
Helium-Neon laser also has very little penetration.

Nd:YAG is the most powerful, with a penetration of 2-6mm, as it has a wavelength of 1064nm.

61
Q

The smallest endotracheal tube that can be railroaded over an Aintree Intubation Catheter has an internal diameter of

A. 4.0
B. 5.0
C. 6.0
D. 7.0
E. 8.0

A

Size 7.0

The Tube

The endotracheal tube has a length and diameter. The endotracheal tubes size (“give me a 6.0 tube”) refers to its internal diameter in millimeters (mm). The ETT will typically list both the inner diameter and outer diameter on the tube (for example, a 6.0 endotracheal tube will list both the internal diameter, ID 6.0, and outer diameter, OD 8.8).

62
Q

When performing an erector spinae block in the lumbar region local anaesthetic should be placed

a. Between the fascial plane of erector spinae and rhomboids
b. Posterior to both erector spinae and spinous process
c. Anterior to erector spinae and posterior to transverse process 5th rib
d. Superficial to the infraspinatus fossa
e .Superficial to the lamina

A

Repeat

c. Anterior to erector spinae and posterior to transverse process 5th rib

Midpoint between T5-6
(Usual Incision T4-5, ICC T6)

63
Q

The drug that is LEAST likely to decrease blood flow to the splanchnic circulation is:

a) Noradrenaline
b) Adrenaline
c) Vasopressin
d) Dopamine
e) Phenylephrine

A

d) Dopamine

Dobutamine (β1 and β2), dopexamine (DA1, some β2) and low-dose dopamine (DA1 and DA2, β1 and β2, α1 in high dose) all have vasodilatory effects on the splanchnic circulation, and have been shown to improve markers of perfusion. For many years, low-dose infusions of dopamine were used as a prophylactic and therapy for acute renal failure, using the logic that DA1- and DA2-mediated vasodilation in renal and splanchnic beds would be protective.

https://pubmed.ncbi.nlm.nih.gov/12794401/

63
Q

A randomised control trial is performed on a new antiemetic medication. The rate of nausea in the placebo group is 20% and in the treatment group the rate is 5%. The number needed to treat to prevent nausea with this new drug is

A

NNT = 6 to 7
NNT=1/ARR.
(Absolute Risk Reduction)

ARR = 0.2-0.05 = 0.15
1/0.15 = 6.66

With respect to previous variation of this question:

(base rate is 10%, experimental group is 8% (20% below 10%) therefore 100/ 2 = 50
or 1 divided by risk reduction

population risk = 10/100 patients get PONV
population risk + new antiemetic = 8/100 patients get PONV (8/100 as reduction by 20% with new drug)

RR= 0.10-0.08=0.02
NNT= 1/RR
=1/0.02
=50

64
Q

The cardiac arrhythmia most commonly associated with the chronic use of methadone is:

a) Torsades
b) VF
c) Tachycardia

A

a) Torsades

2ry to prolonged QT leading to R on T
PETKOV

65
Q

A patient is dyspnoeic in the post anaesthesia care unit with oxygen saturations of 94% on 10 litres/min oxygen via face mask. A focused lung ultrasound is performed. The structures
labelled with the white arrows represent

A

See combined deck for multiple Qs on lung ultrasound

66
Q

The maintenance anaesthetic technique that has the lowest environmental impact from
greenhouse gas is

a) sevoflurane
b) desflurane
c) Halothane
d) Ketamine
e) Propofol

A

e) Propofol

https://www.bjanaesthesia.org/article/S0007-0912(20)30547-X/pdf

66
Q

Cryoprecipitate is a concentrated source of all the following EXCEPT

a) Factor I
b) Factor VII
c) Factor VIII
d) VWF
e) Fibronectin

A

b) Factor VII

Redcross:
Cryoprecipitate contains most of the following found in fresh frozen plasma:
1. factor VIII
2. fibrinogen
3. factor XIII
4. von Willebrand factor
5. fibronectin

Prothrombinex-VF® is a lyophilised concentrate of human coagulation factors it contains:

Factors:
II
IX
X
small amount of factor VII.

Also contains:
plasma proteins (human)
Antithrombin III (human)
Heparin sodium (porcine)
Sodium
Phosphate
Citrate
Chloride

https://litfl.com/cryoprecipitate/

Fractionated plasma product consisting of Fibrinogen (Factor I), von Willebrand Factor, Factor VIII, and small amounts of Factor XIII and Fibronectin

https://www.anzca.edu.au/getattachment/9ec71c61-8a66-4f81-b0f8-c87d65e36298/Australasian-Anaesthesia-2023

67
Q

With regard to Donation after Circulatory Determination of Death (DCDD), the maximum
acceptable time from withdrawal of cardio-respiratory support to cold perfusion for liver
donation is

a) 30mins
b) 45 mins
c) 60 mins
d) 90 mins

A

Warm ischaemia time:
- Time from treatment withdrawal to the start of cold perfusion of the donated organs
- Significance is the impact on graft function
- Most important phase of WIT begins when the systolic BP is < 60mmHg
- This includes the waiting period from the absence of circulation to the declaration of death and the time before initiating the flow of cold perfusate through the cannula

Maximum WARM Ischaemia time
- Heart 30 mins
- Liver 30 mins
- Pancreas 30 mins
- Kidney 60 mins
- Lungs 90 mins

Maximum COLD Ischaemia time:
- Heart = 4 hrs
- Lungs = 6-8hrs
- Liver/Pancreas = 12hrs (DBD)/6 hrs (DCD)
- Kidneys = 18hrs (DBD)/ 12 hrs (DCD)

68
Q

According to ANZCA PS54(A), an anaesthetic machine requiring electrical power must, in the event of mains power failure, be able to operate under battery backup power for a minimum of

a) 30 min
b) 60 min
c) 120 min
d) 240 min

A

REPEAT

a) 30 min

If the anaesthesia machine requires electrical power for normal operation, a backup power supply must be a part of the machine and permit normal operation for at least 30 minutes after a mains power supply failure. An alarm must be activated at the time of the mains failure and the state of the reserve power supply must be indicated while it is in use.

https://www.anzca.edu.au/getattachment/f05e02ec-2023-4c50-b57f-9549ea0c4183/PS54(A)-Position-statement-on-the-minimum-safety-requirements-for-anaesthesia-machines-and-workstations-for-clinical-practice-2021#page=

68
Q

When commencing treatment of proximal deep vein thrombosis or pulmonary embolus, factor Xa inhibitors (apixaban, rivaroxaban) are preferred to dabigatran or warfarin because they do not require

a. A need to dose reduce in pregnancy
b. No need to dose reduce in renal failure
c. No need to bridge
d. Need for monitoring
e. Once daily dosing

A

c. No need to bridge

See ETG recommendations

https://www.ahajournals.org/doi/full/10.1161/JAHA.120.017559

69
Q

A thoracic regional technique that will NOT provide analgesia for sternal fractures is a

a. PECS I
b. PECS II
c. Parasternal intercostal nerve block?
d. Transfascial muscle block (can’t remember wording)
e. transverse thoracic plane block

A

a. PECS I

(PECS II Covers SA and will extend to the sternum)

https://www.youtube.com/watch?v=0g_PGLYy0gw

69
Q

A patient with a history of hereditary angioedema presents to the emergency department with difficulty with breathing, abdominal pain and swelling of the face, hands and feet. The most effective therapy for managing this is

a) C1-esterase inhibitor
b) Danazole
c) Cetirizine
d) FFP

A

a) C1-esterase inhibitor

Treatment options:
Plasma derived C1-esterase inhibitor = Berinert/Cinryze,
Androgens = Danazol
B2 Bradykinin Receptor antagonist = Icatibant
FFP.

Danazol (an androgen) is recommended as first line PROPHYLAXIS for planned procedures (need to give for 5-10 days prior and 2-5 days post)

For emergency or high risk procedures C1 esterase inhibitor concentrate (Berinert or Cinryze) is recommended
- give 1 hour before procedure
- more effective than danazol but more expensive

Berinert:
- 20units/kg IV over 10 min
- Symptoms usually stabilise in 30 mins
- 2nd dose uncommon, but may be given 30mins to 2hrs after 1st dose

Icatibant:
- 30mg slow subcut infusion in abdominal area

Due to the risk of precipitating laryngeal oedema, oropharyngeal procedures should usually involve general anaesthesia with endotracheal intubation

Short answer:
- if you have days before surgery increase danazole, if complex surgery increase danazole and give C1Inh
- If you have acute emergency surgery give C1Inh Concentrate (Berinert/Cinryze) before and after
- if you have an acute attack use C1Inh or Bradykinin antagonist (Icatibant)
- If C1 Inh and Bradykinin antagonoist are not available then use FFP but this may worsen the attack due to the presence of C4 in the FFP
- Has Cetirizine been misremembered instead of Cinryze as an option in this question? No it wasn’t
-> adrenaline, steroids, antihistamines have no role in treatment of HAE acute attack

70
Q

Drug classes demonstrated to reduce mortality in chronic heart failure with reduced ejection fraction include all of the following EXCEPT

A. ACE inhibitors
B. Beta blockers
C. Angiotensin receptor blockers
D. Spironolactone
E. Digoxin

A

Digoxin

71
Q

In the three-bottle chest drainage system set up shown, the maximum suction pressure (cmH2O) generated inside the underwater seal bottle would be minus

A

Depth of tube in water in bottle 3

72
Q

Somatic pain in the second stage of labour is NOT transmitted via the

a) Pudendal nerve
b) Illioinguinal
c) pelvic splanchnic
d) genitofemoral

A

c) pelvic splanchnic
-> visceral not somativ nerve

72
Q

Despite two separate 300 IU/kg doses of heparin, you have failed to attain your target
activated clotting time prior to instituting cardiopulmonary bypass. An appropriate option now would be to give

a. More heparin
b. FFP
c. Dalteparin
d. bivalirudin

A

b. FFP

REPEAT

73
Q

A 35-year-old woman is brought to the emergency department following a suspected
amitriptyline overdose. She has a Glasgow Coma Scale score of 6 and her blood pressure is 90/46 mmHg. Her electrocardiogram is most likely to show

A. AF
B. CHB
C. Sinus tachy with prolonged QRS
D. Sinus brady with prolonged QRS
E. VT

A

REPEAT
c. sinus tachy with prolonged QRS

73
Q

During a thyroidectomy, the surgeon is concerned the parathyroid glands have been
devascularised. From the time of potential damage, a serum calcium level should be checked in

a) 6hrs
b) 12hrs
c) 24 hrs
d) 36hrs

A

24hrs

Oxford handbook

74
Q

A patient with a perioperative troponin rise above the upper limit of normal, chest pain, left ventricular anterior regional wall motion abnormality, and atheroma with a partially occluding thrombus of the left anterior descending coronary artery has had a/an

A) Type 1 MI
B) Type 2 MI
C) NSTEMI
D) MINS

A

A) Type 1 MI

Clinical classification based on the assumed proximate cause of the MI:
- Type 1
○ MI caused by atherothrombotic coronary artery disease
○ And usually precipitated by atherosclerotic plaque disruption ( rupture or erosion)
- Type 2
○ MI consequent to a mismatch between oxygen supply and demand
○ Multiple potentional mechanisms:
§ Coronary dissection
§ Vasospasm
§ Emboli
§ Microvascular dysfunction
§ Increases in demand without underlying coronary artery disease
- Type 3
○ Patient with typical presentation of MI (ECG changes or VF) with unexpected death before blood samples for biomarkers could be drawn
- Type 4a
○ MI associated with Percutaneous Coronary intervion (PCI)
- Type 4b
○ Subcategory of PCI related MI related to stent/scaffold thrombosis
- Type 5
CABG related MI

75
Q

A patient requires elective surgery under general anaesthesia with neuromuscular relaxation.
The recommended preoperative management of donepezil is to

a) cease day before
b) cease 2 weeks before
c) Cease day of surgery
d) continue

A

d) continue

to avoid cognitive decline post-op

Donepezil is in a class of medications called cholinesterase inhibitors. It improves mental function

https://www.ukcpa-periophandbook.co.uk/medicine-monographs/donepezil

76
Q

Cyclooxygenase type 2 inhibitors (COX-2) in pregnancy are considered

a. Not safe
b. safe
c. safe only in 1st trimester
d. safe only in 1st and 3rd trimester
e. not safe for 3rd trimester and 48 hours post delivery

A

a. Not safe
or
c. safe only in 1st trimester

While relatively safe in early and mid pregnancy, NSAIDs can precipitate fetal cardiac and renal complications in late pregnancy, as well as interfere with fetal brain development and the production of amniotic fluid; they should be discontinued in gestational wk 32

APMSE

77
Q

A 75 year-old patient is given a Fleet® sodium phosphate enema prior to a colonoscopy. The hyperphosphataemia from the laxative can directly cause

a) renal failure
b) cardiac failure
c) Arrhythmia
d) severe sleep apnoea

A

a) renal failure

‘…phosphate containing laxatives can lead to acute phosphate nephropathy’
https://academic.oup.com/bjaed/article/16/9/305/1743822#35669023 - BJA Ed article

Phosphate binds to calcium leading to crystal calcium phosphate deposition in tubules.

Old repeat 2020

https://academic.oup.com/bjaed/article/16/9/305/1743822#35669023

78
Q

The QRS axis of the attached electrocardiograph is closest to

A
78
Q

The electrolyte abnormality most associated with an increased risk of laryngospasm is

a. Hypokalaemia
b. Hyponatraemia
c. Hypocalcaemia
d. Hypercalcaemia
e. Hypernatraemia

A

c. Hypocalcaemia

Laryngospasm is a rare, but serious and potentially lethal, complication of hypocalcemia in adults. In every adult presenting with acute dyspnea and stridor, the possibility of hypocalcemia should be considered. Hypocalcemia should be treated promptly.

79
Q

An anaesthetic drug that is safe to use for a patient with porphyria is

a) propofol
b) ketamine
c) thiopentone
d) etomidate

A

a) propofol

79
Q

A patient has received high dose hydroxycobalamin for refractory vasoplegia post cardiac
surgery. Observed effects include all of the following EXCEPT

a) leukopenia
b) red urine
c) falsly low SpO2
d) thrombocytosis

A

c) falsly low SpO2

Effects of hihg dose hydroxycobalamin:
- red urine
- thrombocytosis
- leukopenia

80
Q

The bioavailability of an oral dose of ketamine is approximately

A. 10%
B. 20%
C. 40%
D. 70%
E. 80%

A

B. 20%

25% (a few studies have higher ranges but typically around 20-25%)

https://doi.org/10.1192/bjp.bp.115.165498

80
Q

A venturi mask delivers a fraction of inspired oxygen of 0.28 at the recommended fresh gas flow rate of 6 litres per minute. Increasing the flow rate to 12 litres per minute will deliver a fraction of inspired oxygen of

a) 0.24
b) 0.28
c) 0.36
d) 0.40

A

b) 0.28

81
Q

An inverted u wave is an electrocardiographic sign of

a) Hypokalaemia
b) Raised ICP
c) Digoxin treatment
d) Myocardial ischaemia

A

D> Myocardial ischaemia

An inverted U wave may represent myocardial ischemia (and especially appears to have a high positive predictive accuracy for left anterior descending coronary artery disease[7] ) or left ventricular volume overload.
^Wikipedia
——–
U-wave inversion is abnormal (in leads with upright T waves)
A negative U wave is highly specific for the presence of heart disease
Common causes of inverted U waves

Coronary artery disease
Hypertension
Valvular heart disease
Congenital heart disease
Cardiomyopathy
Hyperthyroidism
In patients presenting with chest pain, inverted U waves:

Are a very specific sign of myocardial ischaemia
May be the earliest marker of unstable angina and evolving myocardial infarction
Have been shown to predict a ≥ 75% stenosis of the LAD / LMCA and the presence of left ventricular dysfunction
^LITFL: https://litfl.com/u-wave-ecg-library/

82
Q

The use of direct oral anticoagulants [DOAC] in atrial fibrillation is contraindicated in the
presence of

a) Bioprosthetic Heart Valve
b) Mitral Regurgitation
c) mild hepatorenal impairment
d) Mitral Stenosis, moderate to severe

A

D) Mitral Stenosis (Rheumatic, moderate to severe)

DOAC use is contraindicated in certain clinical conditions, notably, in patients who have a mechanical heart valve and those with rheumatic mitral stenosis. Moderate to severe renal impairment or significant hepatic disease is also a contraindication to DOAC treatment

Bioprosthetic valves are less thrombogenic thus DOAC use is acceptable. https://www.ahajournals.org/doi/epdf/10.1161/JAHA.120.017559

83
Q

A 30 year old parturient presents in labour. She has a history of Addison’s disease from
autoimmune adrenalitis and has been taking prednisolone 6 mg daily for ten years. On
presentation the patient is given hydrocortisone 100 mg intravenously. The most appropriate steroid replacement regime the patient should receive during labour is

a. 25mg TDS hydrocortisone
b. 8mg/hr hydrocortisone
c. 6mg PO prednisone

A

8mg/hr

Guidelines for mx of glucocorticoids during the perioperative period for patients with adrenal insufficiency

https://associationofanaesthetists-publications.onlinelibrary.wiley.com/doi/10.1111/anae.14963

84
Q

One metabolic equivalent (MET) is equal to

a. O2 consumption walking 4km/h
b. O2 consumption when sitting
c. Energy expenditure walking 4km/h
d. Energy expenditure when sitting.

A

b) O2 consumption when sitting

One metabolic equivalent (MET) is defined as the amount of oxygen consumed while sitting at rest and is equal to 3.5 ml O2 per kg body weight x min.

85
Q

A 64 year old man presenting for elective surgery is on thyroxine 100 mcg daily. His thyroid function tests are: (test results shown) These results are most consistent with:

TFTs thryoxine TSH < .05 T4 and T3 completely normal

a) Hypophysectomy
b) Subclinical Hyperthyoirdism
c) Sick euthyroid
d) Toxic Multinodular goitre

or

a) Overtreatment
b) Subclin hypo
c) Sick euthyroid
d) Falconi syndrome

A

b) Subclinical Hyperthyoirdism
Subclinical hyperthyroidism: low TSH, normal T3 + T4
Clinical hyperthyroidism: low TSH, high T3, high/normal T4

Subclinical hypothyroidism: high TSH, normal T3 + T4
Clinical hypothyroidism: high TSH, low/normal T3, i T4

Amiodarone: high/normal TSH, low T3 (2o to inhibition of pituitary T4 to T3 conversion)

Sick euthyroid: low TSH, low T3

Hypophysectomy (central hypothyroidism): low/normal TSH/T3/T4

Compliant on thyroxine: normal TSH, high/normal T3, low T4
Non-compliant w thyroxine (pt taking several tabs prior to Dr’s appointment): high TSH, normal T4

85
Q

A 42-year-old female is admitted with subarachnoid haemorrhage. She has a severe
headache, has eyes open spontaneously, and is confused but is obeying commands. She is unable to move her left side. The World Federation of Neurological Surgeons grade is

a) 1
b) 2
c) 3
d) 4
e) 5

A

C:3 (Pt is GCS 14 E4V4M6, with motor deficit)

The WFNS scale:

Grade 1: GCS 15, no motor deficit.
Grade 2: GCS 13-14 without deficit
Grade 3: GCS 13-14 with focal neurological deficit
Grade 4: GCS 7-12, with or without deficit.
Grade 5: GCS <7 , with or without deficit.

(BJA Education, Deranged Physiology)

86
Q

Hepatopulmonary syndrome can be treated with

a) Methylene blue
b) Inhaled nitric oxide
c) Nitric oxide inhibitors
d) Oxygen therapy
e) Liver transplantation

A

e) Liver transplantation

  • Oxygen therapy for symptom relief
  • Liver transplant provides long term survival benefit
  • All other therapies tried but no conclusive evidence of benefit/nil are FDA approved

Hepatopulmonary Syndrome Article https://www.ncbi.nlm.nih.gov/books/NBK562169/

Hepatopulmonary syndrome (BJA)
- Prevalence up to 20% (end stage liver disease)
- Characterised by: disordered pulmonary capillary vasodilation and VQ mismatch
- Present with hypoxia, ortheodeoxia (decrease in PaO2 when standing)
- Diagnosis w/bubble echocardiography
- Risk factor for early post-transplant mortality
- If transplant successful, will resolve over time

87
Q

A 30-year-old previously healthy woman is four days post-caesarean section. You are asked to see her to manage her abdominal pain. Over the last two days she has had increasing abdominal pain, increasing abdominal distension, tachycardia and nausea. An abdominal x-ray shows a caecal diameter of 9 cm. After excluding mechanical obstruction, an appropriate management option is:

a) Neostigmine
b) Lactulose
c) Fibre
d) Antispasmotic oral

or

a) neostigmine infusion
b) morphine PCA
c) Naloxone
d) Lactulose

A

a) neostigmine infusion

Consider this Ogilve’s Syndrome
Psuedo-obstruction.
If > 9cm dilation, would need surgical management.

https://www.ncbi.nlm.nih.gov/pmc/articles/PMC3168359/#!po=17.5000

88
Q

The most effective treatment for pain following wisdom teeth extraction as a single oral dose is

a) Paracetamol 1000mg
b) Tramadol 100mg
c) Parecoxib 40mg
d) Ibuprofen 400mg
e) Codeine 30mg

A

d) Ibuprofen 400mg
- Ibuprofen (I think, because of the single oral dose statement)

APMSE 5th edition

Acute pain after third molar extraction is the most extensively studied model for testing postoperative analgesics in single-dose investigations. Nonselective NSAIDs or coxibs are recommended as “first-line” analgesics following third molar extraction (Derry 2011 Level I, 155 RCTs, n=16,104), however paracetamol is also safe and effective with a dose of 1,000 mg providing better pain relief than lower doses (Weil 2007 Level I [Cochrane], 21 RCTs, n=1,968). The best available evidence suggests the use of NSAIDs either with or without paracetamol is effective and well-tolerated (Moore 2018 Level I, 5 SRs, n unspecified).

Nonselective NSAIDs are more effective than paracetamol or codeine (either alone or in combination) (Ahmad 1997 Level I, 33 RCTs, n=5,171). Ibuprofen (200–512 mg) specifically is superior to paracetamol (600–1,000 mg) in this setting and combining these two drugs improves analgesia further (Bailey 2014 Level I [Cochrane], 7 RCTs, n=2,241)

89
Q

The maximum recommended cuff inflation pressure for the classic LMA is

a 15 cm H20
b 30 cm H20
c 40 cm H20
d 60cm H2O

A

d 60cm H2O

90
Q

You have induced a 20-year-old male for appendicectomy with propofol, fentanyl and suxamethonium. You are maintaining anaesthesia with oxygen, air and sevoflurane. His heart rate has climbed to 150 /minute, the ETCO2 is 50 mmHg and his temperature is 40°C.

After turning off the sevoflurane, you should

a) Commence TIVA
b) Give dantrolene 2.5mg/kg
c) Allocate task cards
d) Start active cooling
e) Remove vaporiser

A

e) Remove vaporiser

https://anaesthetists.org/Portals/0/PDFs/Guidelines%20PDFs/Guideline%20Malignant%20hyperthermia%202020.pdf?ver=2021-01-13-144236-793

as per guidelines, see link and attached image

As per anaesthetic crisis manual
1. Call for help, communicate and delegate
2. Stop any volatile and remove vaporiser
3. Allocated task cards
4. Give dantrolene
5. Hyperventilate with 100% high flow oxygen
6. Use activated charcoal filters on both limbs
7. Maintain anaesthesia with TIVA
8. Insert IAL +/- CVC
9. Actively cool if temperature > 38.5
10. Treat associated hyperkalaemia, acidosis, arrhythmias

90
Q

When using the ECG to time intra-aortic balloon counterpulsation, balloon deflation should occur at the

a. start of T wave
b. peak of T wave
c. end of T wave
d. end of R wave
e. peak of R wave

A

e. peak of R wave

https://derangedphysiology.com/main/required-reading/cardiothoracic-intensive-care/Chapter%20634/normal-iabp-waveform

https://litfl.com/intra-aortic-balloon-pump-trouble-shooting/

91
Q

Individuals with Prader-Willi syndrome having an anaesthetic are at most risk of

a) Hypocalcaemia
b) Hypoglycaemia
c) Neuroleptic malignant syndrome
d) Malignant hyperthermia
e) Hypothermia

A

b) Hypoglycaemia

Stoelting:
Prader-Willi syndrome is a rare genetic disorder characterized by
hypothalamic-pituitary abnormalities with severe hypotonia during the neonatal period and
during the first two years of life, hyperphagia with a risk of morbid obesity during infancy and
adulthood, learning difficulties and behavioural problems or severe psychiatric problems. The
disease affects 1/25,000 births.

https://www.orphananesthesia.eu/en/rare-diseases/published-guidelines/prader-willi-syndrome/1339-prader-willi-syndrome-2/file.html

Prader-Willi syndrome is a rare genetic disorder characterized by
hypothalamic-pituitary abnormalities with severe hypotonia during the neonatal period and
during the first two years of life, hyperphagia with a risk of morbid obesity during infancy and
adulthood, learning difficulties and behavioural problems or severe psychiatric problems. The
disease affects 1/25,000 births.

92
Q

Cross clamping of the descending aorta is NOT expected to cause
(MADE UP ANSWERS)

a) Bacterial translocation
b) Decreased Renal perfusion
c) Abdominal compartment syndrome
d) Organ ischaemia
e) Decreased afterload

A

e) decreased afterload

https://academic.oup.com/bjaed/article/13/6/208/246828#2904603

Aortic cross-clamping and physiological considerations

The physiological effect of aortic cross-clamping during surgery varies with the level of the clamp in relation to the main aortic branches. Perfusion to the lower half of the body is therefore dependent on collateral circulation while the clamp is applied.

Clamp application increases the afterload of the heart and a sudden increase in arterial pressure proximal to the clamp; this can be attenuated with vasodilators [e.g. glyceryl trinitrate (GTN), sodium nitroprusside], opioids, or deepening of anaesthesia. These measures may also allow fluid loading in preparation for clamp release; however, the effect of vasoactive drugs is unpredictable; they may change haemodynamics without improving cardiac output and tissue perfusion due to blood redistribution.10

Increased afterload and left ventricular end-diastolic volume both increase myocardial contractility and oxygen demand. This increase in myocardial oxygen demand is usually met by an increase in coronary blood flow and oxygen supply, but can cause myocardial ischaemia.

After aortic cross-clamp release, peripheral vascular resistance decreases by 70–80%, causing a decrease in arterial pressure. Hypotension can also be caused by blood sequestration in the lower half of the body, ischaemia–reperfusion injury, and the washout of anaerobic metabolites causing metabolic (lactic) acidosis. This can cause direct myocardial suppression and profound peripheral vasodilatation. Coronary blood flow and left ventricular end-diastolic volume also decrease (almost 50% from pre-clamp levels) after clamp release.

Strategies to manage hypotension after aortic cross-clamp release include gradual release of the clamp, volume loading, vasoconstrictors, or positive inotropic drugs (e.g. ephedrine, meteraminol, phenylephrine, epinephrine, and norepinephrine). It is important to be aware that vasoactive drugs should only be used after adequate volume repletion.10 Management of aortic cross-clamp application and release requires excellent communication with the surgeon in order to anticipate and manage the physiological effects.

93
Q

Of the following, the LEAST likely to occur during one-lung ventilation in the lateral decubitus position is

a. Intrapulmonary shunt
b. V/Q mismatch
c. Hypercarbia
d. Hypoxia
e. Hypoxic pulmonary vasoconstriction

A

c. Hypercarbia

Single-lung ventilation leads to a right-to-left intrapulmonary shunt as the nondependent lung continues to undergo perfusion with no ventilation, leading to a widened alveolar-to-arterial (A-a) oxygen gradient, which may contribute further to hypoxemia.

Factors leading to decreased blood flow to the ventilated lung also lead to hypoxemia.
Such factors include:
Low Fio2 leads to hypoxic pulmonary vasoconstriction in the dependent ventilated lung
High mean airway pressures in the dependent ventilated lung Vasoconstrictor agents
Intrinsic PEEP

The lateral decubitus position under anesthesia: Under anesthesia, there is a decrease in functional residual capacity. The upper lobe moves under anesthesia to a more favorable portion of the compliance curve versus the lower lung, which lies now on a less favorable portion of the compliance curve. Neuromuscular blockade contributes to abdominal contents pressing against the dependent hemidiaphragm, thereby restricting ventilation. Open non-dependent lung leads to variation in compliance and thus worsens ventilation-perfusion (V/Q) mismatch - thereby leading to hypoxemia. **Carbon dioxide elimination is usually unaffected **in using single-lung ventilation with adequate maintenance of minute ventilation. Both lungs may be affected independently by single-lung ventilation. The ventilated-dependent lung is prone to ventilator-induced lung injury due to higher tidal volumes used. The nondependent nonventilated lung is prone to injury by surgical trauma and ischemia-reperfusion injuries. Considering these physiological changes in single-lung ventilation is vital to safely performing the anesthetic technique and airway management.

Reference: StatPearls Single-Lung Ventilation https://www.ncbi.nlm.nih.gov/books/NBK538314/”

93
Q

Refeeding syndrome following the commencement of total parenteral nutrition is associated with the development of

A

Most likely answer will be related to hypophosphataemia

Refeeding syndrome is a constellation of biochemical abnormalities which occurs when normal intake is resumed after a period of starvation. Its characteristic features are low levels of phosphate, potassium, magnesium and sodium. Its major complications include cardiac arrhythmias, heart failure (due to hypophosphataemia), muscle weakness, rhabdomyolysis, seizures and an altered sensorium.

The major risk factors are calorie malnutrition of any cause, alcohol or drug use, low BMI (18-16) and starvation for 5-10 days.

Pathophysiology
With the restoration of glucose as a substrate, insulin levels rise and cause cellular uptake of these ions. Depletion of adenosine triphosphate (ATP) and 2,3-diphosphoglyceric acid (2,3-DPG) results in tissue hypoxia and failure of cellular energy metabolism. This may manifest as cardiac and respiratory failure, with paraesthesiae and seizures also reported. Thiamine deficiency may also play a part.

  • Exogenous sources of phosphate are inadequate to supplement the daily phosphate requirements
  • Intracellular phosphate stores are used to synthesise ATP (using protein and fat as fuel)
  • Homeostatic mechanisms maintain serum concentrations of these ions at the expense of intracellular stores

Reference: https://derangedphysiology.com/main/required-reading/endocrinology-metabolism-and-nutrition/Chapter%20315/refeeding-syndrome “

94
Q

The peak effect of intravenous insulin on serum potassium when treating hyperkalaemia
occurs at approximately

A. 2 mins
B. 4 mins
C. 10 mins
D. 20 mins
E. 30 mins

A

**D. 20 mins

**The time taken to reduce K+ with insulin/dextrose ranges from ~15-30 mins depending on source
**
https://www.uptodate.com/contents/treatment-and-prevention-of-hyperkalemia-in-adults Treatment approach to hyperkalemic emergencies — Patients with a hyperkalemic emergency should receive (table 1): Intravenous calcium and insulin are rapidly acting treatments that provide time for the initiation of therapies that remove the excess potassium from the body. ●Intravenous calcium to antagonize the membrane actions of hyperkalemia
●Intravenous insulin (typically given with intravenous glucose) to drive extracellular potassium into cells ●Therapy to rapidly remove excess potassium from the body (ie, loop or thiazide diuretics if renal function is not severely impaired, a gastrointestinal cation exchanger, and/or dialysis [preferably hemodialysis] if renal function is severely impaired)
●Treatment of reversible causes of hyperkalemia, such as correcting hypovolemia and discontinuing drugs that increase the serum potassium (eg, nonsteroidal anti-inflammatory drugs, inhibitors of the renin-angiotensin-aldosterone system)

RCH: http://www.rch.org.au/clinicalguide/guideline_index/Hyperkalaemia/ Insulin/glucose to be given at the same time

If severe hyperkalaemia:
- Dextrose 10% : 5ml/kg IV bolus (if no hyponatremia)
- Insulin short action: 0.1 U/kg IV bolus (Max 10 units) Then followed by infusion insulin/glucose (see below)

-If moderate hyperkalaemia:
- Dextrose 10% IV at maintenance with 0.9% sodium chloride (normal saline)
- Insulin short action infusion : 0.1 U/kg/h IV
Note: Close monitoring of glucose every 30-60 minutes
Onset of Action: 15 minutes, should reduce intravascular K+, reduction of 0.5-1.5mmol/L
Duration: peak 60 minutes, 2-3hours

American College of Emergency Physicians:
Nebulized albuterol by face mask begins to take measurable effect after 15 to 20 minutes and lowers the serum potassium level by up to 1 mEq/L, depending on the dose. β-Agonists are safe despite the side effect of tachycardia.

**Insulin, given intravenously in combination with glucose, also results in a similar fall in the potassium level after 20 to 30 minutes **and also lowers levels by up to 1 mEq/L. The combination of nebulized albuterol and intravenous insulin with glucose appears to be additive, lowering serum potassium by a mean of 1.21 mEq/L or more.11

Adult hyperkalemic patients who have ECG changes should receive continuous nebulized albuterol and 50 grams of intravenous dextrose plus 10 units of intravenous regular insulin.

Emergency Medicine Journal J Accid Emerg Med. 2000 May; 17(3): 188–191.
The management of hyperkalaemia in the emergency department

INSULIN WITH GLUCOSE
Insulin binds to specific membrane receptors and via an unknown second messenger, stimulates the sodium-potassium (Na-K) adenosine triphosphatase (ATP) pump resulting in intracellular uptake of K.5 This effect is independent of its hypoglycaemic action. Uraemia attenuates the hypoglycaemic response to insulin but does not affect its hypokalaemic action. Insulin has been the traditional temporising treatment against which newer treatments are compared. It is indicated in every case of hyperkalaemia that needs emergency treatment. Ten units (in adults) soluble insulin is given with 40–60 g glucose intravenously as a bolus. In children, a glucose load of 0.5 g/kg/h (2.5 ml/kg/h) should be given. This is because many of these patients increase their endogenous insulin production with the administration of a glucose load. If the blood glucose rises above 10 mml/l, insulin should be added at 0.05 u/kg/h.24 These studies show that the onset of hypokalaemic action is within 15 minutes and lasts for at least 60 minutes. The reduction in K observed is 0.65–1.0 mmol/l.5, 10 Delayed (30–60 minutes post insulin) hypoglycaemia is common (up to 75% of patients10) if less than 30 g glucose is given.

LITFL: Treatment of hyperkalaemia involves stabilizing the myocardium to prevent arrhythmias, shifting potassium back into the intracellular space and removing excess potassium from the body.

Drive Potassium into the Cell:
Insulin & Glucose
- Dose: IV fast acting insulin (actrapid) 10-20 units and glucose/dextrose 50g 25-50ml
- Insulin drives potassium into cells and administering glucose prevents hypoglycaemia.
- Begins to work in 20-30mins reduces potassium by 1mmol/L and ECG changes within the first hour Ca gluconate
- should be part of initial treatment but it does not lower either total body or serum potassium, it acts as a membrane stabiliser

LITFL: Correct Serious Conduction Abnormalities (Calcium)
- Calcium is a very useful agent. It does not lower the serum potassium level, but instead is used to stabilise the myocardium, as a temporising measure. Calcium is indicated if there is widening of QRS, sine wave pattern (when S and T waves merge together), or in hyperkalaemic cardiac arrest.
- The ‘cardiac membrane stabilising effects’ take about 15-30mins.

Calcium Chloride
- Dose: Calcium Chloride 10% 5-10mL = 6.8 mmol - 3 x more potent than Calcium Gluconate
- Complication: severe thrombophlebitis

  • Calcium Gluconate:
  • Dose: Calcium Gluconate 10% 5-10mL = 2.2 mmol
  • Less potent, less irritating to veins
  • Potential Complications of Calcium administration - Bradycardia, hypotension and peripheral vasodilation
  • Generally these occur if administered too quickly
  • Avoid in digoxin toxicity (use magnesium as alternative)
  • salbutamol
    Drive Potassium into the Cell: Salbutamol
  • Dose: 10-20mg via nebulizer - Beta 2 agonist therapy lower K via either IV or nebulizer route.
  • Salbutamol can lower potassium level 1mmol/L in about 30 minutes, and maintain it for up to 2 hours.
  • Very effective in renal patients that are fluid overloaded
  • Drive Potassium into the Cell: Sodium Bicarbonate
  • Dose: 50- 200mmol of 8.4% Sodium Bicarbonate
  • Bicarbonate is only effective at driving Potassium intracellullarly if the patient is acidotic
  • Begins working in 30-60 minutes and continues to work for several hours.

Eliminate Potassium From the Body: Calcium Resonium
- Dose: 15-45g orally or rectally, mixed with sorbitol or lactulose
- Calcium polystyrene sulfonate is a large insoluble molecule that binds potassium in the large intestine, where it is excreted in faeces
- Effects take 2-3 hours

M&M 2016: An intravenous infusion of glucose and insulin (30–50 g of glucose with 10 units of insulin) is also effective in promoting cellular uptake of potassium and lowering plasma [K+], but may take up to 1 h for peak effect

95
Q

A relative contraindication to a peribulbar needle technique for cataract surgery is:

a) Axial length of 24mm
b) INR 2.5 for mechanical aortic valve
c) Staphyloma
d) Scleral buckle
e) Pterygium

A

c) Staphyloma

https://eyewiki.aao.org/Ocular_Anesthesia#cite_note-:2-3

Contraindications

Absolute
Confirmed allergy to a necessary anesthetic, and nystagmus. Other contraindications are just those of the particular surgery that is to be performed.

Relative
Long eye (in the anterior to posterior axis, evidenced by high myopia), staphyloma (abnormal protrusion at a weak spot in the wall of the eye), enophthalmos (posteriorly or deep set eyes), and extended surgery duration are relative contraindications to retrobulbar and peribulbar anesthesia. In an uncooperative patient, patients deemed to be unable to follow commands during surgery, children and those with uncontrollable neurological movements, general anesthesia may be considered.

96
Q

This arterial blood gas is consistent with a diagnosis of

A
96
Q

Assuming a blood volume of 70 ml/kg, a massive transfusion in a 20 kg, 5-year-old child is
defined as a three-hour packed red blood cell (PRBC) transfusion volume of

a) 250ml
b) 500ml
c) 700ml
d) 1000ml

A

700ml
50% of blood volume in 3 hours

S Blaine. BJAE Paediatric massive transfusion

97
Q

According to the Association of Anaesthetists of Great Britain and Ireland (AAGBI)
guidelines, an acceptable reason to delay surgery in a patient with a fractured neck of femur is

A

https://associationofanaesthetists-publications.onlinelibrary.wiley.com/doi/10.1111/anae.15291

The 2011 guidelines list seven ‘acceptable’ reasons for delaying surgery:
1 Haemoglobin < 80 g.l−1
2 Plasma sodium concentration < 120 or > 150 mmol.l−1 and potassium concentration < 2.8 or > 6.0 mmol−1
3 Uncontrolled diabetes
4 Uncontrolled or acute onset left ventricular failure.
5 Correctable cardiac arrhythmia with a ventricular rate > 120.min−1
6 Chest infection with sepsis
7 Reversible coagulopathy

98
Q

Local anaesthetic blockade of the musculocutaneous nerve in the upper limb will result in
weakness of

A

All muscles in the anterior compartment of the arm are innervated by the musculocutaneous nerve.
- biceps brachii: forearm flexion and supination. Accessory shoulder flexor
- coracobrachialis: shoulder flexion, arm adduction.
- Brachialis: forearm flexion

The musculocutaneous nerve innervates skin on the anterolateral side of the forearm.

99
Q

In patients with symptomatic carotid stenosis, carotid endarterectomy can be performed
within two weeks of initial symptoms if there is/are

a) large stroke area
b) crescendo TIA symptoms
c) haemodynamic instability
d) Tandem Stenosis
e) contralateral occlusion

A

b) crescendo TIA symptoms

https://academic.oup.com/bja/article/99/1/119/269458

Ideally, patients presenting with a suspected TIA should have undergone investigation and, if appropriate, surgery within 2 weeks of presentation.

100
Q

Complications of hyperbaric oxygen therapy do NOT include

a) Myopia
b) Central retinal occlusion
c) Seizures
d) Hypoglycaemia
e) Bradycardia

A

b) Central retinal occlusion

SE’s from HBOT:
- progressive myopia (reversible)
- seizures
- hypoglycaemia
- sinus bradycardia from stimulation of vagal activity bassociated with hyperbaric pressures

101
Q

Of the following, the drug most likely to cause pulmonary arterial vasodilation with systemic arterial vasoconstriction when used in low doses is

a) Adrenaline
b) Noradrenaline
c) Vasopressin
d) Dopamine
e) Dobutamine

A

c) Vasopressin

https://emcrit.org/ibcc/pressors/

  • From UP TO DATE:
    > At low doses of 1 to 3 mcg/kg per min, dopamine acts primarily on dopamine-1 receptors to dilate the renal and mesenteric artery beds
    > At 3 to 10 mcg/kg per min (and perhaps also at lower doses), dopamine also stimulates beta-1 adrenergic receptors and increases cardiac output, predominantly by increasing stroke volume with variable effects on heart rate.
    > At medium-to-high doses, dopamine also stimulates alpha-adrenergic receptors, although a small study suggested that renal arterial vasodilation and improvement in cardiac output may persist as the dopamine dose is titrated up to 10 mcg/kg per min
    *clinically, the haemodynamic effects of dopamine demonstrate individual variability

Dobutamine (inodilator):
- selective β1-agonist that increases cardiac contractility and reduces pulmonary vascular and systemic vascular resistances

Vasopressin:
- vasopressin may have pulmonary vasodilatory effects in addition to a systemic vasoconstrictive effect

Milrinone (inodilator):
- the phosphodiesterase-3 inhibitors, milrinone and enxoimone, have positive inotropic effects combined with the capacity to reduce RV afterload (‘inodilators’) without significant chronotropic effect, but they can be associated with significant systemic hypotension

102
Q

The option below which ranks these pressures from highest to lowest is (atm = atmosphere, cmH2O = centimetres of water, kPa = kilopascals, mmHg = millimetres of mercury, psi = pounds per square inch)

10 atm, 10 cmH2O, 10kPa, 10mmHg, 10PSI

A

All People Kick My Cat
Atm> PSI > KPA > mmHg > cmH2O

1ATM = 14.69 PSI = 101.325 kPa = 760mmHg = 1033 cmH20

102
Q

A patient taking tranylcypromine, a monoamine oxidase inhibitor, requires elective surgery.
The best management is to

(made up answers)

a) Cease 1 month before surgery
b) Do not Cease
c) Cease day of surgery
d) Cease 2 weeks before surgery
e) stop 2 weeks before, start moclobemide and omit Moclobemide day of surgery

A

e) stop 2 weeks before, start moclobemide and omit Moclobemide day of surgery
-> probably in discussion with the patients psychiatrist

Tranylcypromine, sold under the brand name Parnate among others, is a monoamine oxidase inhibitor. More specifically, tranylcypromine acts as nonselective and irreversible inhibitor of the enzyme monoamine oxidase.

In the elective setting, there is some debate regarding the management of patients on MAOI. Although the risks associated with anaesthesia in those taking this group of drugs are significant, abrupt withdrawal may precipitate serious psychiatric relapse. Traditionally, irreversible MAOIs have been stopped 2 weeks before operation; however, omitting the dose of moclobemide on the day of surgery is acceptable. It has been suggested that in the elective situation, patients could be switched from an irreversible MAOI to moclobemide to avoid a prolonged period of discontinuation.

103
Q

A 25-year-old woman is administered two doses of aprepitant for postoperative nausea and vomiting after a sleeve gastrectomy. She normally takes the oral contraceptive pill. You should advise her to use alternative contraception for the next

a) 3 days
b) 7 days
c) 14 days
d) 28 days

A

28 days

Aprepitant PI:
“Alternative or “back-up” measures of contraception should be used during treatment with this medicine and for one month following the last dose of this medicine.”

Pharmacokinetics:
- aprepitant is a CYP3A4 inhibitor
- caution is also advised with warfarin and phenytoin use

104
Q

A four-year-old boy with a history of waddling gait, larger than normal calves and frequent
falls receives a spontaneously breathing volatile-based anaesthetic with sevoflurane. One
hour into the case he develops peaked T waves and then the end-tidal CO2 begins to rise.
The most appropriate immediate treatment is to

a. Temp probe, and go from there
b. Cool + dantrolene
c. Stop volatile, cool + dantrolene
d. Stop volatile, calcium
e. Stop volatile

A

d. Stop volatile, calcium

?Duchenne muscular dystrophy?
This patient most likely has Anaesthesia Induced Rhabdomyolysis (AIR) given the peaked Twaves and slow rise in ETCO2

Immediate MH Management:
Stop administering Sevo, flush machine (or new), charcoal filters. Dantrolene.

105
Q

Jet ventilation for shared airway surgery is traditionally delivered at pressures in atmospheres (atm) of

a) 1 ATM
b) 2 ATM
c) 3 ATM
d) 4 ATM

A

b) 2 ATM

https://academic.oup.com/bjaed/article/7/1/2/509371

**A typical parameter-set for HFJV via a subglottic catheter is DP, 2 atm; f, 150 min−1; Fio2, 1.0; I-time, 50%.
**
Driving pressure 1-2 atm
(250-500ml/s)
RR 8-10

Automated jet ventilator – typical starting jet pressure for an adult is 1.5 bar (~1.5 atm).
Manual jet ventilators deliver up to 3.5-4 bar.

106
Q

A 46-year-old woman with menorrhagia is booked for abdominal hysterectomy. Her
preoperative bloods show

creatinine 55
Ca2+ 2.2
PO43- 0.34.

The most likely reason for these findings is

a) Diuretic use
b) Fanconi syndrome
c) Hyperparathyrodisim
d) Vit D deficiency
a) Iron transfusion

A

a) Iron transfusion

Iron infusion (ferric carboxymaltose) – can cause renal wasting of phosphate resulting in severe hypophosphataemia

Vitamin D deficiency and hyperparathyroidism can also cause hypophosphataemia. Vitamin D deficiency would result in low calcium whereas hyperparathyroidism would result in hypercalcaemia.
Fanconi syndrome: rare defect of proximal tubule leading to decreased reabsorption -> results in hypokalaemia, hypophosphataemia, hyperchloraemic metabolic acidosis. https://www.ncbi.nlm.nih.gov/pmc/articles/PMC6689119/

107
Q

The BALANCED Anaesthesia Study compared older patients having deep anaesthesia
(bispectral index target of 35) to lighter anaesthesia (bispectral index target of 50). It
assessed postoperative mortality and a substudy assessed postoperative delirium. These
showed that, compared to light anaesthesia, deep anaesthesia causes

a) Decreased mortality, no change in post op delirium (POD)
b) No change mortality, reduced POD
c) Decreased mortality, reduced POD
d) No change in Mortality, no change in POD

A

No change in Mortality, no change in POD

No evidence was found that mortality or serious complication were modified by targeting either a BIS of 50 or 35

A broad range of anaesthetic depth can be delivered safely when using volatile anaesthetic agents and processed electroencephalographic monitoring

https://www.thebottomline.org.uk/summaries/pom/balance/

108
Q

A 58-year-old man with ischaemic cardiomyopathy is undergoing a ventricular tachycardia ablation procedure in the catheter laboratory. Partway through the procedure his systolic blood pressure abruptly falls from 110 mmHg to 50 mmHg. The most likely cause for his hypotension is

a) Tamponade
b) RV failure
c) Arrhythmia
d) Anaphylaxis
e) Oesophageal aortic fistula

A

a) Tamponade

Cardiac tamponade occurs ~1%. Can usually be managed with reversal of anticoagulation and percutaneous drainage.

Vascular complications most common followed by tamponade.

https://www.ahajournals.org/doi/10.1161/circep.113.000768

https://academic.oup.com/bjaed/article/12/5/230/289246#3659733

109
Q

A 25-year-old male has continued post operative bleeding after an extraction of an impacted
third molar tooth under a general anaesthetic. The patient mentions that his father bruises
quite easily. His coagulation screen reveals (test results shown). The most likely diagnosis is

His coagulation screen reveals: Prolonged APTT, Normal PT.

a) Factor V Leiden
b) Haemophilia A
c) Haemophilia B
d) Von willebrand disease

A

d) Von willebrand disease
- autosomal dominant inheritance
- may have normal or prolonged APTT, PT is normal

*Haem A: X-linked recessive disorder; would expect prolonged aPTT, and normal PT
*Haem B: X-linked recessive disorder; would expect normal aPTT and normal PT

Up to date:
Inheritance patterns — Most cases of VWD are transmitted as an autosomal dominant trait; this includes types 1 and 2B, and most types 2A and 2M.

Baseline hemostasis assessment —
Most patients will have a complete blood count (CBC) with platelet count and coagulation studies during the initial evaluation for excessive bleeding or bruising.
●Individuals with VWD generally have a normal CBC and a normal platelet count, with the exception of those with type 2B VWD, most of whom will have mild thrombocytopenia (eg, platelet count 100,000 to 140,000/microL).
●Individuals with VWD may have a normal or prolonged activated partial thromboplastin time (aPTT), depending on the degree of reduction of the factor VIII level. The prothrombin time (PT) is normal in VWD.

Up to date:
●Hemophilia A – Inherited deficiency of factor VIII (factor 8 [F8]); an X-linked recessive disorder.
●Hemophilia B – Inherited deficiency of factor IX (factor 9 [F9]); also called Christmas disease; an X-linked recessive disorder.

Laboratory findings —
Hemophilia is characterized by a prolonged activated partial thromboplastin time (aPTT).
However, the aPTT may be normal in individuals with milder factor deficiencies (eg, factor activity level >15 percent), especially in hemophilia B (factor IX deficiency), where even individuals with moderate disease may have a normal aPTT.
In some individuals with hemophilia A, factor VIII levels may increase with stress, leading to a normalization of the aPTT or mis-categorization of factor levels and disease severity.
In patients with hemophilia, the aPTT corrects in mixing studies, unless an inhibitor is present, which only applies to individuals who have received factor infusions or who have an autoantibody such as a lupus anticoagulant or an acquired factor inhibitor.
Mixing studies that do not show correction of a prolonged aPTT suggest an alternative diagnosis such as an acquired factor inhibitor.
The platelet count and prothrombin time (PT) are normal in hemophilia.
Thrombocytopenia and/or prolonged PT suggest another diagnosis instead of (or in addition to) hemophilia.
Measurement of the factor activity level (factor VIII in hemophilia A; factor IX in hemophilia B) shows a reduced level compared with controls (generally <40 percent).
One exception is an individual with mild hemophilia A who undergoes testing when stressed or pregnant and has a falsely elevated factor level. If this is suspected, factor activity testing should be repeated under conditions of low stress.
The plasma von Willebrand factor antigen (VWF:Ag) is normal in hemophilia.
If VWF:Ag is reduced, this suggests the possibility of von Willebrand disease (VWD) rather than (or in addition to) hemophilia.
Urinalysis is not done routinely, but if performed it may sometimes (but not always) show microscopic or macroscopic hematuria.

109
Q

The coronary artery most likely occluded in this ECG of an acute ST-elevation myocardial
infarction is the

a) RCA
b) L Cx
c) LAD
d) Left Main CA
e) Posterior Descending CA

A

c) LAD

110
Q

According to the Fourth Consensus Guidelines for the Management of Post-operative
Nausea and Vomiting (PONV) published in 2020, multimodal PONV prophylaxis should be
implemented in adult patients

a. For everyone
b. 1 or more RF
c. 2 or more RF
d. 3 or more RF
e. 4 or more RF

A

b) 1 or more RF

111
Q

The most appropriate initial diagnostic test for a suspected phaeochromocytoma is a/an

A

serum free metanephrines and nor-metanephrines

https://www.ncbi.nlm.nih.gov/pmc/articles/PMC3230088/

https://www1.racgp.org.au/ajgp/2021/january-february/adrenal-disease-an-update

112
Q

An adult patient is administered a target controlled propofol infusion for more than 30
minutes with a constant effect-site target of 4 mcg/ml propofol plasma concentration.
Compared to the Schnider model, the propofol dose given by the Eleveld model will be a

a) Smaller bolus lower infusion rate
b) Smaller bolus hihger infusion rate
c) Larger bolus lower infusion rate
d) Larger bolus highier infusion rate
e) Smaller bolus same infusion rate

A

c) Larger bolus lower infusion rate

https://associationofanaesthetists-publications.onlinelibrary.wiley.com/doi/10.1111/anae.13345

https://journals.lww.com/anesthesia-analgesia/fulltext/2014/06000/a_general_purpose_pharmacokinetic_model_for.12.aspx

112
Q

A 4 week old full term neonate with an inguinal hernia, who is otherwise healthy, has an ASA (American Society of Anesthesiologists) classification of at least

a) 1
b) 2
c) 3
d) 4

A

ASA 3

112
Q

You are inserting a pulmonary artery catheter in an intubated patient prior to cardiac surgery, and a significant amount of blood appears in the endotracheal tube. The most appropriate specific initial management is to

A. Remove PAC and insert DLT
B. Wedge PAC and insert DLT
C. Wedge PAC and insert bronchial blocker
D. Withdraw PAC 2 cm and insert DLT
E. Withdraw PAC and insert bronchial blocker
F. Inflate balloon

A

D. Withdraw PAC 2 cm and insert DLT

LITFL: Pulmonary haemorrhage after PAOP measurement

a life threatening time-critical emergency
pulmonary artery rupture caused by inflation of the pulmonary artery catheter (PAC) balloon during ‘wedging’ (measurement of the pulmonary artery occlusion pressure)
some experts advise against measuring PAWP because of the risk of pulmonary artery rupture
0.2% risk,  30% mortality

RISK FACTORS

pulmonary hypertension
mitral valve disease
anticoagulants
age >60 years

MANAGEMENT

Goals

prevent further pulmonary haemorrhage
stop bleeding
resuscitate

Call for help

ICU consultant
anaesthetist/ OT
cardiothoracic surgeon
interventional radiology

Resuscitation

A
    may have to be emergently intubated if not already
B
    FiO2 1.0
    controlled ventilation
    if able to recognize which lung is haemorrhaging may be able to perform lung isolation (insert single lumen tube into unaffected side, exchange for a double lumen tube or use bronchial blocker with bronchoscopic assistance)
    apply PEEP to tamponade wound
C
    large bore IV cannulae, fluids, blood products, inotropes

Specific therapy

Lay the patient ruptured side down
withdraw pulmonary catheter 2-3 cm with balloon down then refloat PAC with balloon inflated to occlude pulmonary artery (to try to tamponade bleeding)
stop antiplatelet agents and anticoagulants
give reversal agents:
— protamine for heparin
— platelets for anti-platelet agents
give blood products as indicated by FBC, coags and clinical state
interventions
— angiogram or bronchoscopy to isolate pulmonary vessel involved
— if bleeding doesn’t settle will require lobectomy
113
Q

This patient has been requested to look straight ahead. He is suffering from a right

a) Horner’s Syndrome
b) 3rd nerve palsy
c) 4th nerve palsy
d) 6th nerve palsy

A

b) 3rd nerve palsy

https://derangedphysiology.com/main/required-reading/neurology-and-neurosurgery/Chapter%204631/lesions-oculomotor-nerve-cn-iii
This is the “down and out” eye syndrome. It is characterised by ptosis, a down-and-out pupil, mydriasis, absent light reflex with intact consensual constriction of the opposite eye, and failure of accommodation. Classically, this is the lesion which develops during uncal herneation, due to an ipsilateral cerebral injury.

Causes of unilateral CN III lesions:
- Uncal herneation: Pressure from herniating uncus on nerve
- Fracture involving ipsilateral cavernous sinus
- Cavernous sinus thrombosis (ipsilateral)
- Aneurysm (ipsilateral)
- Midbrain lesion (see Question 26.2 from the second paper of 2011)

Causes of bilateral CN III lesions:
- Cavernous sinus thrombosis
- Aneurysm
- Contralateral brainstem lesion (midbrain)

Exclusion of a 4th nerve lesion
- Tilt the head to the same side as the lesion
- The affected eye will intort if the fourth nerve is intact.

113
Q

A baby is brought to the emergency department three days after a term home birth. It has not been feeding well and has had few wet nappies. The child is grey in appearance and femoral pulses are difficult to palpate. You note an enlarged liver and marked tachycardia. Pulse oximetry reveals saturations of 75% despite oxygen being administered. You suspect a duct- dependent circulation. The best initial management is:

a) Adrenaline
b) Noradrenaline
c) IV Fluid 20ml/kg
d) Alprostadil (PGE1) infusion
e) Intubation and controlled ventillation

A

Alprostadil (PGE1)

From Paediatric BASIC on CHD:
- Resuscitation of an infant or newborn in shock should follow a standard approach regardless of the aetiology.
- Any patient with a duct dependent lesion either for pulmonary blood flow, or systemic output, will require PGE1. The problem is that whether or not a duct dependent lesion is present is unclear in most cases. If CHD has been diagnosed antenatally, PGE1 should be started.
- The cyanosed neonate presenting with severe cyanosis (O2 <75) and/or in extremis should be started on PGE1; the assumption being that the duct has closed and needs to be reopened.

114
Q

The antiemetic least likely to precipitate an arrhythmia in a patient with this ECG is

a) Droperidol
b) Metoclopramide
c) Promethazine
d) Dexamethasone
e) Ondansetron

A

d) Dexamethasone
The ECG shows LONG QT

https://litfl.com/qt-interval-ecg-library/

114
Q

A woman with atrial fibrillation has no valvular heart disease. According to AHA guidelines,
oral anticoagulants are definitely recommended if her CHA2DS2-VASc score is greater than or equal to

A. 1
B. 2
C. 3
D. 4
E. 5

A

C. 3

  • if male CHA2DS2-VASc score ≥2 to be recommended chronic OAC (Grade 1A).
  • if female CHA2DS2-VASc score ≥3 to be recommended chronic OAC (Grade 1A).
  • non-sex risk factor also holds bearing:
  • For patients with CHA2DS2-VASc score of 1 in males and 2 in females based on age 65 to 74 years, we recommend chronic OAC (Grade 1A).

Up to date:

Our approach to deciding whether to prescribe anticoagulant therapy for patients with AF (excluding those with rheumatic mitral stenosis that is severe or clinically significant [mitral valve area ≤1.5 cm2], a bioprosthetic valve [surgical or bioprosthetic] within the first three to six months after implantation, or a mechanical heart valve) is as follows:

*For a CHA2DS2-VASc score ≥2 in males or ≥3 in females, we recommend chronic OAC (Grade 1A).

*For a CHA2DS2-VASc score of 1 in males and 2 in females:
-For patients with CHA2DS2-VASc score of 1 in males and 2 in females based on age 65 to 74 years, we recommend chronic OAC (Grade 1A). Age 65 to 74 years is a stronger risk factor than the other factors conferring one CHA2DS2-VASc score point.
-For patients with other risk factors, the decision to anticoagulate is based upon the specific nonsex risk factor and the burden of AF. For patients with very low burden of AF (eg, AF that is well documented as limited to an isolated episode that may have been due to a reversible cause such as recent surgery, heavy alcohol ingestion, or sleep deprivation), it may be reasonable to forgo chronic OAC and institute close surveillance for recurrent AF, although it may not be possible to reliably estimate AF burden from surveying symptoms or infrequent monitoring. The frequency and duration of AF episodes vary widely over time, and episodes are often asymptomatic.

*For patients with a CHA2DS2-VASc of 0 in males or 1 in females, we suggest against OAC (Grade 2C). Patient values and preferences may impact the decision. For example, a patient who is particularly stroke averse and is not at increased risk for bleeding may reasonably choose anticoagulation, particularly if the patient is a candidate for treatment with a direct oral anticoagulant (DOAC).

2019 AHA/ACC/HRS Focused Update of the 2014 AHA/ACC/HRS Guideline

114
Q

The CRASH-2 trial showed tranexamic acid administration to trauma victims results in a
reduction in

a. Decreased mortality
b. Increased mortality
c. Decreased blood product use
d. No change mortality
e. Increased bleeding

A

a. Decreased mortality

Death in bleeding trauma patients
Early administration of TXA safely reduced the risk of death in bleeding trauma patients and is highly cost-effective. Treatment beyond 3 hours of injury is unlikely to be effective.

  • Reduced death due to bleeding x 0.85
  • Equivocal blood transfusion
  • Equivocal thromboembolism
115
Q

Of the following, the lowest level at which neurogenic shock is likely if an acute spinal cord
injury were to occur at that level is

a) C2
b) C6
b) T4
c) T6
d) T10

A

c) T6

LITFL: https://litfl.com/trauma-spinal-injury/

Neurogenic shock is classically characterised by hypotension, bradycardia and peripheral vasodilatation. Neurogenic shock is due to loss of sympathetic vascular tone and happens only after a significant proportion of the sympathetic nervous system has been damaged – as may occur with lesions at the T6 level or higher.

Spinal shock is not a true form of shock. It refers to the flaccid areflexia that may occur after spinal cord injury, and may last hours to weeks. It may be thought of as ‘concussion’ of the spinal cord and resolves as soft tissue swelling improves. Priapism may be present.